123
Torts Torts Introduction: 1. Three categories of torts a. Intentional torts : where the defendant desires to bring about a particular harmful result b. Negligence : defendant has not intended to bring about a certain result, but has merely behaved carelessly c. Strict Liability : defendant is held liable even though he did not intend to bring about the undesirable result, and even though he behaved with utmost carefulness i. Conducting abnormally dangerous activities ii. Selling a defective product that causes personal injury or property damage iii. There is no fault in strict liability, this is the only case where accidentally caused harm is also tortious. 2. The more culpable the tortfeasor, the more far-reaching his liability for unexpected consequences. a. Culpability is highest in an intentional tort and least in strict liability. b. The measure of damages is generally broader for the more culpable categories. i. D is more likely to be required to pay punitive damages when he is an intentional tortfeasor than when he is negligent or strictly liable. 3. Three tort scenarios: a. Stranger cases = random interaction between two people without a defined relationship b. Consensual arrangements = where the parties know each other and are in some kind of agreed upon relationship (often basis for contractual relationship) c. Highway (license) cases = where access is limited but interactions are still between strangers 4. The interests protected by tort law: a. Physical security and autonomy of person and property i. Physical harms are ordinarily tortious in the absence of justification b. Emotional security and other intangible interest such as privacy and reputation i. Emotional harms may or may not be held tortious. Courts are more reluctant to impose liability. 1

Torts - University of Chicagoblsa.uchicago.edu/first year/TORTS/Torts-Epstein2002b.doc · Web viewTorts Introduction: Three categories of torts Intentional torts: where the defendant

  • Upload
    buiminh

  • View
    226

  • Download
    0

Embed Size (px)

Citation preview

Page 1: Torts - University of Chicagoblsa.uchicago.edu/first year/TORTS/Torts-Epstein2002b.doc · Web viewTorts Introduction: Three categories of torts Intentional torts: where the defendant

Torts

TortsIntroduction:

1. Three categories of tortsa. Intentional torts : where the defendant desires to bring about a particular harmful

resultb. Negligence : defendant has not intended to bring about a certain result, but has

merely behaved carelesslyc. Strict Liability : defendant is held liable even though he did not intend to bring about

the undesirable result, and even though he behaved with utmost carefulnessi. Conducting abnormally dangerous activities

ii. Selling a defective product that causes personal injury or property damageiii. There is no fault in strict liability, this is the only case where accidentally

caused harm is also tortious.2. The more culpable the tortfeasor, the more far-reaching his liability for unexpected

consequences. a. Culpability is highest in an intentional tort and least in strict liability.b. The measure of damages is generally broader for the more culpable categories.

i. D is more likely to be required to pay punitive damages when he is an intentional tortfeasor than when he is negligent or strictly liable.

3. Three tort scenarios:a. Stranger cases = random interaction between two people without a defined

relationshipb. Consensual arrangements = where the parties know each other and are in some

kind of agreed upon relationship (often basis for contractual relationship)c. Highway (license) cases = where access is limited but interactions are still between

strangers4. The interests protected by tort law:

a. Physical security and autonomy of person and propertyi. Physical harms are ordinarily tortious in the absence of justification

b. Emotional security and other intangible interest such as privacy and reputationi. Emotional harms may or may not be held tortious. Courts are more reluctant to impose

liability.c. Economic security and opportunity

i. Where there is no physical harm, courts are cautious about imposing liability for purely economic harms.

5. Aims of tort law:a. Compensation of injured personsb. Deterrence of undesirable behaviorc. Encouragement of social utility and personal responsibilityd. Risk distribution

1

Page 2: Torts - University of Chicagoblsa.uchicago.edu/first year/TORTS/Torts-Epstein2002b.doc · Web viewTorts Introduction: Three categories of torts Intentional torts: where the defendant

Torts

Intentional TortsLiability is based on intent to commit tort.

A. Trespassory torts accomplished through use of force1. Trespassory torts are regarded as harmful in themselves

a. P does not have to prove physical/emotional harm b/c it will be assumed once tort is proven

b. Cf. negligence where P always has to prove actual harm2. Rationale:

a. These torts impact P’s need for security in person and propertyb. Impair P’s rights of autonomy and self-determination

B. Intent1. Intent is tort specific, but D must have

a. Purpose to accomplish a specific resultb. Substantial certainty that the action will bring about a specific result

i. Mere risk, even a high risk, that a result will occur is not sufficient to show substantial certainty

ii. If it is merely "highly likely," not "substantially certain," that the bad consequences will occur, then the act is not an intentional tort. "Recklessness" by D is not enough.

iii. Substantial certainty must relate to actions affecting a particular P or Ps at a particular time and place and caused by a particular source of harm

iv. If D knows with substantial certainty that a particular effect will occur as a result of her action, she is deemed to have intended that result

Garratt v Dailey (WA 1955): D, 5 year old boy, pulls a chair out from under arthritic old P as she is sitting down. If D acted maliciously and knowing with "substantial certainty" that P would hit the ground, D meets the intent requirement for battery, even if he did not desire that she do so. (Indirect causation case: D didn’t touch P, but he made it impossible for her to avoid falling.)

2. Intent is specific: D has to have purpose or substantial certainty to accomplish one or more specific objectives

a. Example: D can intend to touch P and intend to harm P—these are two different intents, and the intent to touch is not necessarily tortious.

b. Three aspects of intent: Intent to do action Intend action to have effect Act occurs

The prima facie case does not require showing intent to harm but just showing action. Brown v Dellinger (TX Civ App 1962): Two children held liable for fire damage to house b/c they had trespassed into garage and lit fire, fire spread accidentally from the grill.

3. Intent is subjective: in question is the state of mind of the D.a. Cf. negligence where standard is objectiveb. Evidence of intent is objective: based on D’s actions

4. Intent is not dependent upon bad motive5. With infants and mentally disabled or insane, question is whether they had formed the

required intenta. McGuire v Almy (MA 1937): In 1930 P hired to work as nurse for insane old woman. P lived-in,

sleeping in room next to D’s room. D was sometimes violent. In 1932, D became very violent, destroyed furniture in her room. P knew D was dangerous, worried D would hurt herself. P called D’s

2

Page 3: Torts - University of Chicagoblsa.uchicago.edu/first year/TORTS/Torts-Epstein2002b.doc · Web viewTorts Introduction: Three categories of torts Intentional torts: where the defendant

Torts

brother-in-law and maid, then opened door and approached D who struck and injured P. Held: don’t want innocent victim to suffer. RULE: where an insane person intentionally does harm, he is liable for his tort. Codicil: insane person must have been capable of entertaining intent and must have entertained it in fact.

b. Polmatier v Russ (Conn 1988): Schizophrenic beat his father-in-law to death, tried to argue he couldn’t form requisite intention to be liable. Court didn’t accept this defense, though D found unfit to stand criminal trial. Held liable for intentional tort. Rational choice by insane not required since they can still have intent, even if the intent is not rational.

c. Rationale: don’t want innocent victim to suffer, want estate of insane person to pay in order to reduce incentive for normal people to pretend they are insane in order to profit from their actions.

6. Intent versus negligence—mutually exclusivea. Negligence=unreasonably risky conduct determined by what a reasonable person

would find unreasonably risky.b. Do not need to show D’s purpose or intent to create a risk or to have harm result—

state of mind is not at issue, but only outward conduct. Distinguish D’s act from the consequences of that act.

The act must be intentional or substantially certain, but the consequences need not be. Once actor deliberately causes contact, the tort is complete and the actor is liable for all the consequences, even if they were not what he intended.

If D intends only minor harm but causes serious harm, the law still holds him liable for the harm caused. Anyone who deliberately violates the bodily [or property] space of another takes the risk that his actions will produce harm greater or different from the one he intended and he can’t then try to limit his liability by pointing to his original intent to do less harm than occurred. Cleveland Park Club v Perry (D.C. 1960): Boy damages club pool by inserting ball into pipe, thinking there was no suction. Boy was not trespasser at pool. Ruling: the intent controlling is the intent to complete the physical act and not the intent to cause injurious consequences. Boy did not intend for ball to be sucked up into pipe, but he was acting in excess of the permission he had on premises.

When is lack of intent still liable? (Trespass, mistake, transferred intent) Mistake as to consequences:

D flips switch intending to turn on light, but instead releases gas which causes injuries.

D might be innocent b/c of disconnect between his intent and the unexpected consequences or D might be liable if he had made unauthorized use of P’s property

Transferred intent: if D held the necessary intent with respect to person A, he will be held to have committed an intentional tort against any other person who happens to be injured

(As long as the actual victim did not assume the risk of accidental consequences, e.g. by trespassing on D’s land) Talmage v Smith (MI 1894): P struck in eye and injured by stick D threw at others who were trespassing on D’s land. D did not intend to hit P, did not even see P. D liable b/c he intended to harm someone.

Summary: Intention in social setting among familiar parties: okay to intend to touch as long as there is no intent to offend or harm.

3

Page 4: Torts - University of Chicagoblsa.uchicago.edu/first year/TORTS/Torts-Epstein2002b.doc · Web viewTorts Introduction: Three categories of torts Intentional torts: where the defendant

Torts

Intention in stranger cases: it is only necessary to show that D acted at all, not that D intented to harm.

Vosburg v Putney (WI 1891): D lightly kicked P in shin in classroom. This resulted a day later in permanent injury to Ps leg, perhaps due to aggravation of an earlier injury. Trial courts found for P b/c D kicked P, kick caused injury, even though D did not intend to injure P. B/c special verdict found that D did not intend to harm P, court made up new rule that if act was unlawful, the intention to commit it is unlawful. Act was unlawful b/c it occurred in classroom

Prima facie case of intentional tort: intent only important to distinguish between act of D and natural event acting on D’s body.

B. Battery1. Definition: The intentional infliction of a harmful or offensive bodily contact.2. Intent: intentionally causing bodily contact to P in a way that is:

a) Harmfulb) Not justified by privilege or by P’s apparent wishesc) The touching is in fact harmful or against P’s wishes.

i. An accidental touching may be negligent, but it is not intentional.3. It is not necessary that D desires to physically harm P as long as touching was intentional

and not privileged.4. Contact can be harmful or offensive, i.e. damaging to a reasonable sense of dignity

a) Alcorn v Mitchell (IL 1872): P and D were litigants in court. After decision, D spit in face of P in court. P sues for offensive conduct. (Court has to give damages to prevent such acts which are provocative of violence in order to preserve public tranquility.)

5. Contact can occur via direct touching, using some instrumentality to harm P, including e.g setting a tripwire.

a) Touching objects intimately associated with P’s body can also be contact, e.g. pulling lapel, knocking off hat.

b) Respublica v De Longchamps (PA 1784): D struck cane of French ambassador which is considered enough for offensive battery b/c it’s an insult and b/c protection against offensive battery extends to anything closely attached to the P’s person.

6. It is not necessary that P have actual awareness of the contact at the time it occurs.a) Example : D kisses P while P is asleep.

When is intent not intent? (reciprocal touching, inevitable touching) Reciprocity and social custom: allowable level of touching—there is no intent to harm,

offend, or embarrass. In social settings, social convention allow people to touch without liability for harm

caused Actor is still not liable if the act would be considered socially acceptable by a reasonable

person, but the victim finds it offensive. unless actor knows that victim would find it offensive implied license of the playground: there is regularity in social interaction and

everyone understands the risks Actor is liable if he acts in a way a reasonable person would find offensive even though

the actor did not realize this or intend to give offense. Tuberville v Savage: “if one strike another upon the hand, or arm, or breast in discourse it is no assault,

there being no intention to assault, but if one intended to assault, strike at another and miss him, this is an assault.”

4

Page 5: Torts - University of Chicagoblsa.uchicago.edu/first year/TORTS/Torts-Epstein2002b.doc · Web viewTorts Introduction: Three categories of torts Intentional torts: where the defendant

Torts

Vosburg v Putney: outcome would have been different had kick occurred on playground b/c there is reciprocity in that situation.

Where contact is inevitable, there is no battery even if the touch leads to serious harm; e.g. jostling on busy street.

Summary of battery D must act: R2T §2: an act is an external manifestation of the actor’s will. D’s act must be intentional—has to be result of a voluntary act

D must act for the purpose of inflicting harmful or offensive contact on P or realize that such contact is substantially certain to result.

The act must cause a contact with P The intended contact must be either harmful or offensive to P

Objective measure of offensive contact: contact that would offend a reasonable sense of personal dignity under the given circumstances. R2T §19.

C. Assault1. Definition: Assault is the intentional causing of an apprehension of immediate harmful or

offensive contact.a) D must (1) act with intent, (2) to place victim in apprehension of a harmful or

offensive contact or to make such contact, (3) the victim must reasonably be placed in apprehension of such contact.

2. Intent—either D intends to create apprehension or D intends but fails to commit battery:a. Intent to create apprehension: D intends to put P in imminent apprehension of the

harmful or offensive contact, even if D does not intend to follow through (e.g., D threatens to shoot P, but does not intend to actually shoot P)

i. Either D acts on purpose or with substantial certainty that apprehension will result, even if he did not act for purpose of making P afraid.

b. Intent to make contact: D intends to in fact cause a harmful or offensive bodily contact, if P is placed in apprehension of the battery. (E.g. D shoots a gun at P, trying to hit him. D hopes P won’t see him, but P does. P is frightened, but the shot misses. This is assault.)

c. It is not necessary that D bear malice towards P, or intend to harm P.i. I de S and Wife v W de S (Eng 1348): D came to P’s closed tavern, got angry, struck door

with his hatchet. P’s wife came to the window and looked out. D saw her, and struck hatchet in her direction, but did not hit her. There was no trespass b/c no harm done, but there was an assault for which damages were assigned. (Important to note that D had already struck door with his hatchet, so P had a reasonable apprehension.)

ii. Allen v Hannaford: P hired moving me to move her furniture. D claimed a lien against the furniture and came to the apt brandishing a gun threatening the movers and P. D found guilty of assault.

d. P has to prove that he reasonably interpreted D’s act as being harmful or offensive. P has to have a reason to have been apprehensive.

e. Assault turns on whether D’s act would place a reasonable person in apprehension of an unwanted contact, not whether D is in fact able to make the threatened contact.

3. P must be aware of the threatened contact.a) P must have an apprehension that she herself will be subjected to a bodily contact.

She may not recover for her apprehension that someone else will be so touched.b) Apprehension versus fear : Apprehension= the perception or anticipation of a battery. P does not

have to be afraid. It is enough the P believes D’s threatened act is capable of immediately inflicting

5

Page 6: Torts - University of Chicagoblsa.uchicago.edu/first year/TORTS/Torts-Epstein2002b.doc · Web viewTorts Introduction: Three categories of torts Intentional torts: where the defendant

Torts

the contact upon him. It doesn’t matter that P could easily prevent the act by self-defense or that P assumes by-standers will intervene. Fear implies a doubt about whether the act can be prevented.

4. Imminence : It must appear to P that the harm being threatened is imminent, and that D has the present ability to carry out the threat.

a) R2T§29 comment b : imminent does not mean instantaneous but rather that there will be no significant delay. It is not necessary that one shall be within striking distance of the other, or that a weapon pointed at the other shall be in a condition for instant discharge. It is enough that one is so close to striking distance that he can reach the other almost at once, or that he can make the weapon ready for discharge in a very short interval of time.

5. Words alone do not suffice to give rise to an assault. There must normally be some overt act by D

a) Brooker v Silverthorne: D threatens phone operator over phone. No assault found b/c threat promises only a future injury, here there was no expression of an intention to injure in the future and therefore no threat.

i. There has to be apprehension of imminent harm, so words spoken at a distance are not assault.

b) Exception : when surrounding circumstances or D’s past actions make it reasonable for P to interpret D’s words alone as creating the required apprehension of imminent contact.

i. Tuberville v Savage (1669): P puts hand on sword and says if it weren’t assize time, I wouldn’t take such language from you. D takes this as an assault and attacks P in what D says was self-defense. Court found no original assault b/c the intention as well as the act makes an assault.

c) Conditional threat : Where D threatens the harm only if P does not obey D’s demands, the existence of an assault depends on whether D had the legal right to compel P to perform the act in question. If there is a legal right, D has committed an assault.i. But: P, a burglar, breaks into D’s house. D says, "If you don’t get out, I’ll throw you out." There

is no assault on P, since D has the legal right to force P to leave.6. One cannot commit an assault against real or personal property.

D. False imprisonment1. Definition : the intentional infliction of a confinement.

a) P’s interest in freedom of locomotion is unconditional, and D cannot put her to a choice between liberty and physical injury, or between her liberty and retention of property, when she is legally entitled to both.

2. P must show that D either intended to confine him, or at least that D knew with substantial certainty that P would be confined by D’s actions.a. The tort of false imprisonment cannot be committed merely by negligent or reckless

acts.i. Example: D, a shopkeeper, negligently locks the store while P, a customer, is in

the bathroom. This is not false imprisonment if harms are minor, since D did not intend to confine P.

ii. But if harms are great, D will be held liable for negligence3. Confinement : The idea of confinement is that P is held within certain limits, not that she is

prevented from entering certain places.a) Example: D refuses to allow P to return to her own home. This is not false

imprisonment – P can go anywhere else, so she has not been confined.

6

Page 7: Torts - University of Chicagoblsa.uchicago.edu/first year/TORTS/Torts-Epstein2002b.doc · Web viewTorts Introduction: Three categories of torts Intentional torts: where the defendant

Torts

b) Bird v Jones (KB 1845): public highway enclosed for race. P wants to go onto that part of the street, but D prevents him. D does not physically constrain him, P is free to go in any other direction. No imprisonment found.

4. The imprisonment may be carried out by direct physical means, but also by threats or by the assertion of legal authority.

a) If P reasonably believed that D has legal authority to confine, even though D doesn’t though he says he does, this is still false imprisonment.

5. P must either be aware of the confinement, or must suffer some actual harm.6. Confinement can be for any duration7. Question of how secure confinement must be: there can be some escape, but P is not

required to run a substantial risk of injury in order to escape, nor does P have to leave property behind.

E. Intentional Infliction of Emotional Distress1. Definition: the intentional or reckless infliction, by extreme and outrageous conduct, of

severe emotional or mental distress, even in the absence of physical harm.a. Cannot be conduct which is merely thoughtless, tasteless, offensive, or inconsiderate. Has to be

extreme behavior intolerable in society.2. Intent: three types of culpability

a. D desires to cause P emotional distress; or,b. D knows with substantial certainty that P will suffer emotional distress; or, c. D recklessly disregards the high probability that emotional distress will occur.

3. Transferred intent: applies only in a very limited fashion for emotion distress torts. E.g. if D attempts to cause emotional distress to X (or to commit some other tort on him), and P suffers emotional distress, P usually will not recover.

a. Exception : transferred intent doctrine is applied if: i. D directs his conduct to a member of P’s immediate family;

ii. P is present; and iii. P’s presence is known to D.

4. P must show that D’s conduct was extreme and outrageous. D’s conduct has to be "beyond all possible bounds of decency.

5. P must suffer severe emotional distress. P must show at least that her distress was severe enough that she sought medical aid. Most cases do not require P to show that the distress resulted in bodily harm.

a. Liability is difficult to establish. Wilkinson v Downton (QB 1897): As practical joke, D tells P that he husband had been in a bad accident and

had broken his legs. Lead to extreme emotional breakdown. Held as tortious b/c D intended to produce a reaction in P that he could have anticipated would be harmful.

George v Jordan Marsh Co (MA 1971): bill collectors harassed P who suffered two heart attacks. Rockhill v Pollard (OR 1971): Ps injured in car accident, taken to nearby doctor, D, who basically ignores

them. Doctor has duty to patients, and this behavior was outrageous.

F. Trespass to land1. Intentional interference with rights of exclusive possession2. Definition: trespass occurs when either:

a. D intentionally enters or causes entry onto P’s land, without permission; b. D remains on P’s land without the right to be there, even if she entered rightfully; or c. D puts an object on (or refuses to remove an object from) P’s land without

permission.

7

Page 8: Torts - University of Chicagoblsa.uchicago.edu/first year/TORTS/Torts-Epstein2002b.doc · Web viewTorts Introduction: Three categories of torts Intentional torts: where the defendant

Torts

3. Trespass refers only to intentional interference with P’s property.a. D is not liable for trespass when he enters by negligence, by accident, or without an

act of his own, e.g. car goes out of control and enters land, badly hit golf ball hits house.

b. D would only be liable if he is shown to have been negligent in causing the harm, but he is not a trespasser b/c intent requirement not met.

4. Physical harm to the land is not required.a. Damage is broadly construed so that at least nominal damages are allowed in almost

any case of entry.b. Since intent t enter is sufficient for the tort, one who intends to enter land in the

mistaken belief it is his is still technically trespassing and liable for any damages.i. Maye v Tappan (CA 1863): D dug up gold-bearing earth from Ps land, after P mistakenly

told D he owned the land.5. Trespass also lies for invasion of land below surface and airspace immediately above it.6. When P authorizes D’s entry, P is held to assume risk of accidental injury but not deliberate

wrong.a. D is not liable for trespass for unintended harm when he is rightfully on the land or

when he enters by negligence, accident, or without acting on his own volition.b. Cleveland Park Club v Perry (D.C. 1960): Boy damages club pool by inserting ball into

pipe, thinking there was no suction. Boy was not trespasser at pool. Ruling: the intent controlling is the intent to complete the physical act and not the intent to cause injurious consequences. Boy did not intend for ball to be sucked up into pipe, but he was acting in excess of the permission he had on premises.

G. Trespass to chattel (Trespass de bonis asportatis)1. Definition: any intentional interference with a person’s use or possession of a chattel in

a way that causes harm. a. D only has to pay damages, not the full value of the property (as in conversion).

2. Intent required is the intent to act upon the property; intent to harm is not necessary.a. But trespass to chattel does not lie where D did not intend to touch chattel but

did so only accidentally3. Differs from trespass to land b/c P must show legally recognizable harm

a. Actual dispossession, orb. Loss of use for an appreciable period, orc. Damage to the chattel, ord. Personal harm to P or to someone or something in which P had a legal interest.

i. Trivial interference does not count as dispossession.ii. Transferred intent: harm necessary to trigger liability can be harm to

something other than the chattel itself. E.g. if by moving a chattel a negligible amount D causes harm to a person, D is liable for the injuries even though the dispossession was trivial.

8

Page 9: Torts - University of Chicagoblsa.uchicago.edu/first year/TORTS/Torts-Epstein2002b.doc · Web viewTorts Introduction: Three categories of torts Intentional torts: where the defendant

Torts

D’s Defensesrelevant to all torts

A. Defenses defeating a claim: 1. action was justified2. action was not tortious at all3. actor was privileged

1. Privilege: exempts a potential D from liability to another.a) D can prove a privilege yet still be held liable for exceeding it, e.g. using more force

than reasonably necessary for self-defense.b) Examples: arrest, imprisonment, surgery

2. Justification: D is believed to have acted in a way that reasonably people would rightly act under the same circumstances.

c) Justifications tend to invoke objective standards of reasonableness to modify the flat rules of trespassory torts.

d) Examples: self-defense, necessity3. Excuse: D does not assert that his action was rightful and that others would act in the same

way, but that D’s action was understandable given his personal condition and that he is therefore not blameworthy under the circumstances.

1. Excuses focus on subjective mental or psychological characteristics of the actor, e.g.:a. mental disability —mentally disabled are subject to liability for their

intentional torts under the same rules that apply to others, but only if they can form intent and act on it.

b. infancy —children old enough to form tortious intent are liable for their intentional torts. If child is too young, this can negate required intent.

c. mistake —can be part of justification where privileges protect the D who acts as a reasonable person under the circumstances. Honest mistake can negate required intent.

2. In general excuses do not furnish defenses to intentional tort claims as justifications do.

B. Consent1. Consent or apparent consent bars P from recovering damages for acts that would otherwise

be tortiousa) Consent = ex ante, so in assessing the liability ex post, don’t consider

consequences.b) Assumption of risk is the analogous rule for negligence

2. P effectively consents if appearances created by her words or acts lead D reasonably to believe that she consented, even if she did not intend to.

a) O’Brien v Cunard SS Co. (MA 1891): immigrant appeared to consent to inoculation.b) Implied consent can arise from custom

i. Example: custom to let drivers turn around in your driveway.3. If P consented to conduct, she cannot recover damages for harm resulting from the

conduct even if P did not expect harm to result and did not consent to harm.4. Duress, fraud, incapacity obviate consent

a) If P mistakenly consents and D knows or should have known about the mistake, consent is obviated.i. Unless the consent is about a collateral matter

9

Page 10: Torts - University of Chicagoblsa.uchicago.edu/first year/TORTS/Torts-Epstein2002b.doc · Web viewTorts Introduction: Three categories of torts Intentional torts: where the defendant

Torts

b) Exception: Substituted consent by guardians for minors and those incapable of giving consent.

c) Sexually transmitted diseases : the defense of consent to physical contact can be overridden if the consent was induced by fraud or even by nondisclosure or some material fact, e.g. that one party had an std.

5. Consent to illegal acts—catch 22: if P’s consent is a bar, D is not liable for injury to P even though it was illegal. If consent is no bar, P can profit from his illegal act by suing D.

a) Allowing P to sue promoter/organizer creates incentive for D to follow the rules.b) R2T §892C: consent is a bar, prosecute D criminally.c) Hudson v Craft (CA 1949): 18 year old solicited to box at carnival, boxing not licensed or run

according to statute. P sues promoter for injuries. Ct allows suit.6. Scope of consent—D is only protected in doing those acts to which P consented.

a) Problem is in determining scope of consenti. Argue the facts, consider the possibility of a contractual solution

b) Mohr v. Williams (MN 1905): P gave D consent to operate on one ear but not the other. D was liable when he operated on the unintended ear.

c) Kennedy v. Parrott (NC 1956): surgeon punctures ovarian cysts while doing appendectomy. P suffers harm from mistakenly cut blood vessel. Held: Dr used sound judgment and P gave implied consent—ct limited consent to place of the incision.

d) Playground/sports cases: Ps accept reciprocal harms and/or harms occurring within the rules of the game, but not harms occurring through D’s recklessness and/or intentionally breaking the rules.i. Hackbart v Cincinnati Bengals (--): P injured by late hit in football game.

7. Emergency rule : in medical emergencies, consent is implied from the circumstances.a) Issue of necessity: the ordinary rules of autonomy and consent are suspended when

there is no opportunity to transact in ways that advance one’s own interests.b) Judge necessity of action not by the outcome but by whether the decision was sound

when it was made.C. Defense of self and others

1. D is privileged to use reasonable force to defend himself or others against unprivileged acts and any threatened confinement or imprisonment that he reasonably believes will cause him bodily harm or offensive bodily contact.

a. The privilege exists even where D’s act would amount to a battery, assault, or false imprisonment

b. D use of force is limited to the degree necessary to prevent the threatened harm as it reasonably appears to D under the circumstances.

i. If D uses more force than necessary, he will be liable for damage caused by the excess.

ii. Deadly force may not be used unless the D is himself in danger of death or serious bodily harm.

2. Self-defense may be used not only where there is a real threat of harm, but also where D reasonably believes that there is one.

3. The defense of self-defense applies only where D uses the force needed to protect himself and others against immediate harm.

a. D may not use any degree of force in retaliation for a tort already committed.b. D may not use force to avoid harm that is not imminent, unless it reasonably

appears that there will not be a later chance to prevent the danger (immediate necessity to prevent a future harm).

10

Page 11: Torts - University of Chicagoblsa.uchicago.edu/first year/TORTS/Torts-Epstein2002b.doc · Web viewTorts Introduction: Three categories of torts Intentional torts: where the defendant

Torts

4. Duty to retreat: R2T holds that a. D may use non-deadly force rather than retreating; but b. D may not use deadly force in lieu of retreating if retreating is safe, except if

attacked in his own dwelling by one who does not reside in the dwelling.c. Many courts allow deadly force to counter deadly force without a general

requirement of retreat.d. If the harm threatened is unintentional, the result of P’s negligence, D should

reasonably avoid the harm if possible5. Standard of measure of self-defense is objective: was it what a reasonable person

would have done under the circumstances.a. An irrational D cannot justify an attack based on his subjective belief that he

was in danger.b. An honest and rational D who responds reasonably though mistakenly is not

subject to liability.i. Courvoisier v Raymond (CO 1896): D’s store/home was invaded by group of men.

He chased them out and they started throwing things at him. D fired gun into air, attracting police. One policeman, P, approached him, D shot him thinking that he was one of the attackers and was threatening him. Have to decide if D’s act was justifiable even if mistaken.

ii. Epstein theory: the guy who makes the mistake bears the consequences; why should innocent P suffer.

1. Roman Law: if you harm an innocent by stander by mistake, you are liable. Self-defense can only be against the person threatening to harm.

E. Necessity:8. A necessity situation is like a monopoly—Trespasser’s options are limited, rules of K

and rights of private property are abridged.9. Under necessity, T has privilege to harm/trespass upon the property of Owner where

this is necessary to prevent great harm to 3rd persons or to T.10. If owner refuses to allow T to enter, and T is harmed, owner is liable for the harm.

a) Ploof v Putnam (VT 1908 ): P sailing on Lake Champlain. D owns island in lake and a dock thereon. Property was run by D’s servant. Storm arises. To save his ship, goods, and family, P docks at D’s dock without permission but under necessity. D’s servant forcefully unmoored ship, which was driven ashore and destroyed.

11. Private necessitya) If T harms O’s property used under necessity, T is liable for the actual damage

causedb) This privilege exists notwithstanding a prior contributory negligence of the

trespasseri. Vincent v Lake Erie Transportation Co (MN1910): D owns steamship, was docked at P’s

dock in order to unload cargo. While unloading an extraordinarily potent storm came up, and by the time unloading was completed, the ship could not safely be moved. The storm caused the ship to be thrown against the dock damaging it and the ropes mooring it to wear away. The ropes were replaced. D was held liable for the damage to the dock.

ii. Vincent outcome can be justified under theory of unjust enrichment why should D be able to save his expensive boat but dock owner have to suffer a loss?

c) T is entitled to use force to obtain the right to use the property if owner tries to resist.

d) The privilege lasts only as long as the necessity.e) Necessity versus self-defense

11

Page 12: Torts - University of Chicagoblsa.uchicago.edu/first year/TORTS/Torts-Epstein2002b.doc · Web viewTorts Introduction: Three categories of torts Intentional torts: where the defendant

Torts

i. Difference is in the source of the threat. With self defense, the source is the party against whom D uses force. With necessity T didn’t create the situation, it is unrelated to the person harmed.

f) Public Necessity:i. when government agents destroy some private property to protect the wider

community interests. The privilege is complete and no action will lie against either the private or government parties whose conduct is justified by a public necessity. Privilege is necessary to protect public official to act without being concerned about ramifications.

F. Defense of property & Recapture of chattel12. RULE: Possessor of land or chattels, when present, may use reasonable force to defend

possession.a) Example: Store may remove unruly customer, even though the touching is batteryb) Example: Land owner may remove trespassing chattel left on land, even though doing so is

trespass to chattel or conversion.13. Less strong a defense for D than defense of persons b/c law afraid of abuses.14. Courts distinguish between defense of property and recovery. Once an owner

completely loses possession of the property—land or chattel—the privilege of defense is exhausted. To recover the property, P must resort to the courts.

15. Possessor may only use self-help proportionate to the threat.a) If P is not damaging D’s property, D cannot use forceb) D cannot use force to prevent theft of property

i. Corollary: D cannot use traps (e.g. spring gun) to protect property from trespassers. R2T §85: use of trap is permissible only when D could have used that force in person.

c) Exception: D can use force to protect his home16. Recapture of land : policy preference that it be done peacefully and not through self-

help.a) cts divided over whether owner can use any force at all in repossessing property.b) Berg v Wiley—landlord changes locks on property when tenant is away as means of peaceable

eviction but court ruled this a tort b/c not peaceable b/c the only reason violence didn’t erupt is b/c tenant was away.

17. Recapture of chattel :a) Privilege allowed when one person wrongfully obtained possession of the

chattel by either force, fraud, or without claim of right.i. Not allowed where D voluntarily parts with his property which is then

retained by P against D’s will. Kirby v Foster (RI 1891): P was accountant for D. $50 had gone missing, so P’s

pay was docked. Later D handed P money to pay workers. P took out $50, handed the rest back to D and said he was taking what was his and leaving. D then used force to try to get the money back. Held: no privilege b/c P did not take property against the will of D, but was given it by D, so the property could not be retaken by force even though P did convert it.

b) Hot pursuit requirement: any privilege of recapture must be exercised promptly, or it is lost.

12

Page 13: Torts - University of Chicagoblsa.uchicago.edu/first year/TORTS/Torts-Epstein2002b.doc · Web viewTorts Introduction: Three categories of torts Intentional torts: where the defendant

Torts

Non-intentional torts: from strict liability to negligence standardA. Accidental harms: standard of liability

a. Negligence : ask whether D has taken the care of a reasonable person, whether by act or omission, in order to avoid the harms that might foreseeably flow from his actions.

i. Ex post standard looking back, did D do all he could?b. Strict Liability : D is liable for harm caused to P though D acted without any wrongful

intent and with all possible care to avoid harm to others.i. Ex ante standard D knows in advance that he will be liable for the results of

his actions.c. Affirmative defenses: assumption of risk, contributory negligence, (inevitable accident)d. Absolute liability—no defenses, no excuses allowed.

B. History: a. Thorns case (1466): D sued for trespass when some of the thorns he was trimming on

his own land fell onto the land of his neighbor and he deliberately entered P’s land to retrieve them, causing damage. D’s entry is deliberate, but he claims a privilege to enter similar to necessity b/c the falling was against his will. Court held D was not excused. Littleton thought liability was strict; Choke raised standard like negligence with high degree of care: would allow privilege if D had done all he could to prevent the thorns falling b/c like act of God; but agreed that cattle straying even against the will of owner would be strict liability.

b. Tithes case (1506): D moved P’s grain to keep it safe but it was then ruined in the new location. Again D’s action was intentional, but the harm was not. D held liable of something akin to strict liability b/c there was no negligence defense allowed that D had done his best to protect the grain, nor was a justification allowed.

c. Weaver v Ward (1616): D shoots P during militia exercise—accidental harm. D allowed no excuse except where the act was not at all due to D’s action (e.g. third person pulls the trigger, act of God, contributory negligence, inevitable accident). Inevitable accident moderates strict liability and opens the door to negligence since D could still be held liable if his own negligence contributed to the ultimate accident like sudden necessity.

d. Smith v Stone (1647)—an exception: third persons carried D onto P’s land, D was utterly passive so not liable for trespass.

e. Gilbert v Stone (1647)—strict liability: D takes P’s horse to escape a mob, but D is still liable for trespass to chattel b/c D was actor and P would have no recourse against the pursuers.

C. Trespass and Case a. Trespass = direct and immediate harmsb. Case = indirect and consequential harms

i. Squib case Scott v Shepherd (1773): demonstrates fuzzy boundary between direct and indirect. The initial throwing of the firecracker was unlawful and would have been clearly trespass had it blown up right then. But other forces intervened. Blackstone thought this made it case, but the other judges held that the throwing and the explosive power were part of the same initial force set in motion by D.

13

Page 14: Torts - University of Chicagoblsa.uchicago.edu/first year/TORTS/Torts-Epstein2002b.doc · Web viewTorts Introduction: Three categories of torts Intentional torts: where the defendant

Torts

ii. Guile v Swan (1822): balloonist lands in garden which gets trampled by onlookers who come to his aide. Action for trespass allowed b/c balloonist directly drew trespassers.

c. Dichotomy breaks down with vicarious liability cases b/c if D’s driver ran down P, P would have to sue driver in trespass and D in case.

i. Williams v Holland (1833) paradigm

Willful NegligentDirect Trespass Trespass or caseIndirect (Scott v Shepherd) Case

ii. After this change, trespass is no longer solely strict liability.iii. Case is preferred as the form of action b/c it covers both direct and indirect

harms, meaning that negligence becomes the dominant standard of liabilityd. After the forms of action are abolished, the more important demarcation between strict

liability and negligence becomes property damage and personal injury respectively.i. strict liabilitythe rule for property damage b/c property not mobile and it

doesn’t create its own mischief. P’s own conduct called into question more in personal injury cases.

1. Negligence standard more favored in personal injury b/c it tends to cut out liability in cases where P’s recovery would be blocked by contributory negligence.

D. Rise of Negligence a. Brown v Kendall (MA 1850): D hit P in eye with a stick D was using to try to separate

D and P’s dogs who were fighting. D backing up, raised stick over his head, P moving forward. P required to show that D had unlawful intent or failed to use reasonable care. If the act of hitting P was unintentional and raising the stick was lawful, then D held not liable unless he acted without due care under the circumstances, i.e. a negligence standard.

b. Resurgence of strict liability in property damage—Rylands v Fletcher (1865-1868): Contractor dug reservoir on D’s land. Under reservoir were mine shafts, leading onto P’s land. Reservoir burst, leading to flooding in P’s mines. Is it direct harm trespass and strict liability or indirect harm case and negligence b/c act was lawful and results against D’s will?

i. Problem in dealing with liquids and gravity, the distinction breaks down. Is it the accumulation of water that causes the harm or the single molecule that, when added, breaks the reservoir?

ii. Blackburn’s Rule : the person who for his own purposes brings on his lands and collects and keeps there anything likely to do mischief if it escapes, must keep it in at his peril, and, if he does not do so, is prima facie answerable for all the damage which is the natural consequence of its escape. He can excuse himself by showing that the escape was owing to the P’s default; or perhaps that the escape was the consequence of vis major, or the act of God.

1. Result: harm was consequential but liability was still strict, reverses Brown v. Kendall outcome—difference is that that was personal injury.

2. Americans didn’t like Rylands—saw it as interfering with industrial progress

14

Page 15: Torts - University of Chicagoblsa.uchicago.edu/first year/TORTS/Torts-Epstein2002b.doc · Web viewTorts Introduction: Three categories of torts Intentional torts: where the defendant

Torts

Brown v Collins (NH 1873): D had horses and wagon halted at train crossing. Train frightened horses, who bolted, ran onto P’s land, struck a light post he owned on the land. D managed horses with ordinary care and skill until they bolted. Criticism of Rylands:

1) anything a person brings onto land outside of the most primitive state is going to be unnatural, so he will have to be concerned about the possible harm of everything. This penalizes development.

2) that possible harm is going to be held against him no matter how remote his fault, e.g. if a spark from his fire travels up his chimney and starts a fire on his neighbor’s roof without any show of negligence

Law has to adapt as society changes. Held: no liability, Rylands doesn’t apply.Losee v Buchanan (NY 1873): Ds boiler exploded and damaged P’s buildings next door. D not liable on theory that everyone benefits from advances in society/technology, so if through no fault or negligence or nuisance my property harms another, there is no liability since the harmed party is also benefiting in general from progress.Turner v Big Lake Oil (TX 1936): rejects Rylands as inapplicable b/c in Texas water cisterns are natural and a necessary and common use of the land.

Powell v Fall (QB 1880): Sparks from a train being properly managed ignite a rack of hay on P’s farm. Statute seems to reserve right of action to injured 3rd party resulting from use of locomotives. Held: use strict liability Rylands standard. If benefit from using train is less than cost of recompensing damage, machine is a nuisance and should not be used.

Strict liability and negligenceOutcome theory strict liability: D knows in advance what unacceptable consequences are

and D figures out how to avoid them.Input theory negligence: D has to be able to foresee specific harms in order to be liable for

them.Stranger cases—involuntary interactions Moral case—corrective justice

Strict liability: comparative approach—as between the two parties it is better that the liability be imposed on the party who caused the harm rather than on the innocent victim who suffered it. Level of care is not the issue: if neither P nor D has been lax, why penalize P? Relevant distinction is not moral blameworthiness but action v. passivity As long as D hopes to gain from his action, it is not fair to foist the costs off onto P Right of individuals to autonomy and use of property should be protected against

incursions. Strict liability treats the invasion as wrongful act requiring redress unless some

justification or excuse is found Negligence: the losses lie where they fall unless and until there is some good reason to

shift them. No liability without fault, where fault is the equivalent of negligence.

As long as D has taken due are, no reason to shift liability onto him. If P would have done as D did, then D is not blameworthy.

Prima facie case for negligence needs harm inflicted + wrongfully. Basic question: did D fail to take care appropriate to his condition and the

circumstances?

15

Page 16: Torts - University of Chicagoblsa.uchicago.edu/first year/TORTS/Torts-Epstein2002b.doc · Web viewTorts Introduction: Three categories of torts Intentional torts: where the defendant

Torts

Sum: strict liability imposes costs on party who seeks the gain; negligence allows a party to seek gain as long as his actions are as careful as those he takes when he bears the risk of loss as well. Both lead to conduct that D would adopt if he were the single owner of all relevant

resources. Use of liability rules as instruments for economic growth

Strict liability imposes burdens but also protects from the harms imposed by others. Negligence imposes constraints b/c each actor has to take the same care as he would

with his own property. No strong correlation between choice of liability rules and level of social wealth or

rate of economic development. Two strategies to meet threat of liability

Reduce risk Insure against risk

Under strict liability, D only reduces risk to point at which insurance is cheaper than prevention.

Economic efficiency Cost-benefit analysis of optimal care relative to cost of precaution. Hand formula: Liability depends upon whether burden of prevention is less than injury

times probability Task of tort law is to find a way to induce D to take the same level of precaution that he

would if he were to suffer the harm himself. Include:

Administrative costs of running tort system Error costs that lead to wrong judgments and dull incentives

Negligence and strict liability produce the same optimal outcome to minimize the sum of accident and prevention costs. Under both systems, D takes cost-justified precautions

Negligence : D takes optimal cost-justified precautions, beyond this he is not liable b/c not negligent so it’s not worth his spending additional money on precautions.

Strict liability : D internalizes costs of all harms avoided and compensates P for all unavoided harms, but the cost of avoiding harms beyond optimal cost-justified precaution point is too expensive, so D ends up at same point as he would under negligence system.

Total efficiency also has to minimize administrative and error costs. Negligence fewer lawsuits b/c of more restrictive conditions for liability, but the

cost of litigation is higher in each case. Have to examine all relevant inputs to find negligence

Strict liability more lawsuits, lower litigation costs. Look only at output and chain of causation

Negligence susceptible to more errors. Bolton v Stone (HL 1951): woman on public sidewalk hit by unusually long-hit cricket ball from nearby

field. Lord Radcliffe: no breach of legal duty unless D’s act fell below reasonable man standard of reasonable/ordinary care. Consider what the reasonable man would have done when confronted with the same remote risk= nothing.

16

Page 17: Torts - University of Chicagoblsa.uchicago.edu/first year/TORTS/Torts-Epstein2002b.doc · Web viewTorts Introduction: Three categories of torts Intentional torts: where the defendant

Torts

Rinaldo v McGovern (NY 1991): inexperienced golfer hits ball into street where it hits P’s windshield. Under negligence D isn’t liable, but that doesn’t provide incentives to take precautions as strict liability would.

Highway cases Stranger cases with added structure set out for all users by third party, usually the state.

Liability not just determined by who hit whom, but also who was complying with the rules of the road.

Issue: what legal regime maximizes benefits to all users? Negligence, strict liability, first-party no fault insurance for all harms except deliberate

or reckless ones? Negligence and strict liability merge b/c driver who fails to comply with rules is prima

facie negligent. But if D is taking ultimate precaution, he would still be strictly liable.

Difficult questions: Nonnegligent rule breaking

Necessity—where D causes harm will seeking to avoid imminent peril Sudden and unanticipated physical or mental incapacities

In most states these conditions excuse liability if party causing harm had no prior warning of the danger.

Hammontree v Jenner (Cal App 1971): Man with history of epilepsy, controlled by medication, has seizure loses control of car, which runs into P’s business. D was doing nothing wrong according to rules of DMV, so no negligence.

Consensual Cases Consensual cases subject to large degree of variability b/c cover many different situations

and relationships. Some differentiation in liability rules may be needed to respond to this diversity Parties may have used contract provisions to deal with liabilities

Basic points: P is at lower risk than with stranger or highway cases b/c P chooses party with whom he

has relationship Liability rules shouldn’t diminish P’s incentive to choose trading partners wisely.

So don’t make P’s recourse against D too easy after the fact In a trading relationship any profits taken by D have to be sufficient to cover anticipated

costs of liability to P in addition to costs of doing business So P pays in advance premium against future liability payments as inducement to D

to do business P’s problem becomes how to maximize anticipated gain from entering into the

original contract. Balance level of protection in event of harm and price which includes liability

premium. Allowing free contracting on liability won’t necessarily have to solve the problem the

way the tort system does workers’ comp system takes whole different approach. Cost of accident prevention high + class of unavoidable accidents large negligence

system preferred (e.g. medical malpractice)

17

Page 18: Torts - University of Chicagoblsa.uchicago.edu/first year/TORTS/Torts-Epstein2002b.doc · Web viewTorts Introduction: Three categories of torts Intentional torts: where the defendant

Torts

Under a strict liability system, P would have to pay such high liability premiums that the cost of the servive would be prohibitive.

Cost of avoidance low + number of unavoidable accidents small strict liability system preferred. (construction defects in products liability, like glass in can of tuna) Precautions can sharply reduce accidents – strict liability makes sense But with products design cases, looks more like medical malpractice negligence

Conclusions Neither negligence nor strict liability dominates entire field of personal or property

injury There are other possible solutions to the liability issue

Hybrids Negligence is not a single standard but a range of them depending on standard of

care required

NegligenceMost areas of tort law are governed by negligence standard

Elements of negligence Duty of reasonable care to P Breach of the duty required of a reasonable person, creating a risk to P Causation—cause in fact & proximate cause Resulting damages suffered by P.

Duty of reasonable care Two categories

Duty to strangers Duty to persons with whom D has special relationship

Reasonableness standard—reasonable person & reasonable care “The omission to do something which a reasonable man guided upon those

considerations which ordinarily regulate the conduct of human affairs, would do, or doing something which a prudent and reasonable man would not do.” Blyth v Birmingham Waterworks

“Conduct which falls below the standard established by law for the protection of others against unreasonable risk of harm” R2T § 282

“unless the actor is a child, the standard of conduct to which he must conform to avoid being negligent is that of a reasonable man under like circumstances.” R2T § 283.

Reasonable person Objective standard (subject to qualifications)

Vaughn v Menlove (1837): D set up hayrick on edge of property, knowing it was dangerous. It spontaneously combusted, burning P’s property. Held: not good enough that D acted to best of his judgment. Had to have a “regard to caution such as a man of ordinary prudence would observe.” The standard is action followed by “the prudent man under the circumstances.”

Note that this is a shift from the strict liability standard found in earlier cases:

18

Page 19: Torts - University of Chicagoblsa.uchicago.edu/first year/TORTS/Torts-Epstein2002b.doc · Web viewTorts Introduction: Three categories of torts Intentional torts: where the defendant

Torts

Beaulieu v Finglam (1401): P’s house burned by fire carelessly watched by D’s servant. Held: owner liable for house guest or licensee, but not for uninvited 3rd party.

Tuberville v Stamp (KB 1697): D so carelessly guarded his fire that sparks blew away and burned P’s field. Held: maxim to use own property so as not to injure another’s is general whether inside or out. It’s his fire, he builds it as his peril. Act of Gd is another issue, but that isn’t shown here.

These are bomb and trigger situations Problem: how must guidance does objective standard actually provide?

Justifications: belief that reasonable man standard doesn’t have to be reconstructed

for every tort. Want to protect P from harms he cannot control, and reasonable care

considered higher standard than good faith judgment. Risk of D’s lack of intelligence or discretion falls on D. Encourages D to take precautions to avoid being put in a situation

he cannot handle. Knowledge : when D is justifiably ignorant of some background information,

this introduces subjective element into standard and makes negligence harder to sustain. But, if D knows activity he is engaged in is potentially dangerous, he has

duty to investigate and take precautions Ignorance is not excuse when knowledge is obtainable at reasonable

cost. Skill : the standard of care is typically not adjusted for levels of ability

beginners are expected to maintain same standard as those who are reasonably skilled Ps should not have to subsidize D’s learning process.

R3T approach: hold beginners to same standard in interactions with strangers; make some allowances for consensual interactions. New driver on road—same standard as all drivers New driver with instructor in parking lot—instructor assumed the risk.

Experts : If a person holds himself out as being more skilled than the average person, he will be held to that higher standard. Rest § 299A

Interaction with others : D cannot cause others to cause harm, e.g. by entrusting dangerous instrument to people not competent to use them (giving guns/explosives to children or insane people)

Infancy : children are given an exception, but not if they are involved in adult activities R2T §283A: holds child to standard of a reasonably careful person of the same

age, intelligence, and experience. Exceptions :

courts divided if children over 14 should be held to age appropriate or adult standards

lower standard of care for infant—R3T: children under 5 incapable of negligence.

Children engaging in adult activities are held to same standard of care as adults (driving, snowmobiling, powerboating, driving farm equipment)

19

Page 20: Torts - University of Chicagoblsa.uchicago.edu/first year/TORTS/Torts-Epstein2002b.doc · Web viewTorts Introduction: Three categories of torts Intentional torts: where the defendant

Torts

Daniels v Evans (NH 1966): P=19 years old, killed when his motorcycle hit by D’s car. Minor has to be judged by standard of care of a minor, not an adult when engaged in the activities of a child, but when engaged in adult activities, like P, has to be held to adult standards of care.

But not applied to hunting accidents involving guns or to skiing, probably not with bicycling on public road.

Old age : highway and stranger cases held to same standard of care as general public. (Ability to conform to traffic rules is prerequisite to using roads, no matter the

age) Roberts v Ring (MN 1919): half-blind old man, with knowledge of his defects,

driving car, hits child who darts out into road. Held to reasonable standard of care.

Insanity : R3T: actor’s mental or emotional illness is not considered in determining whether conduct is negligent.

Except for case of sudden incapacitation or unconsciousness, not reasonably foreseeable when D begins conduct.

Does this exception give wrong incentives not to take precautions when warning signs occur?

Insanity defense limits: Don’t want innocent P to bear the burden Induce those in control of insane’s estate to control the person Fear of false claims

Breunig v American Family Ins (WI 1970): woman hearing voices, drives into oncoming vehicle. Issue: did she have forewarning of insanity. Held, yes, so no defense.

Physical disability : reasonableness is “care which a reasonable person under the same or similar disability would exercise under the circumstances.” Based on fairness and deterrence—don’t want to penalize person for being

blind, but also don’t want him doing things he can’t safely do. P also has to act as a reasonable person

If P knows D has created a danger, P has to take steps to mitigate injury to himself.

When P is child or elderly, P’s unique abilities will be taken into account, since P did not create the danger.

D’s responsibility to disabled P: 1) failure to take precautions needed for ordinary people—and if P is injured

he is not contributorily liable if he took reasonable precautions himself. 2) precautions for foreseeable danger to disabled who are part of public

using public space (location specific). Fletcher v City of Aberdeen (WA 1959): City dug ditch in parking strip to lay wires.

It had set up barriers, but a workman moved barrier and negligently failed to replace it. A blind man, having no forewarning of the ditch, fell in. If the barrier had been in place, he would have been stopped. City has to keep sidewalk in repair for all citizens.

Sudden emergency doctrine : in an emergency a person is not expected or required to use the same judgment and care that is required in calmer moments. If the person acts as a reasonably careful person would act in a similar emergency, there is not negligence even though afterward it might appear that something else would have been safer.

Courts disapprove of the doctrine, b/c standard is built into negligence instruction.

20

Page 21: Torts - University of Chicagoblsa.uchicago.edu/first year/TORTS/Torts-Epstein2002b.doc · Web viewTorts Introduction: Three categories of torts Intentional torts: where the defendant

Torts

Lyons v Midnight Sun Transportation (AK 1996): Woman pulled out in front of a van and was hit and killed. P asserts van driver was driving too fast, and if he had swerved in other direction, P would not have been killed. But held that negligence was not cause of accident and D did his best under the circumstances to avoid accident. Break situation down: antecedent negligence of D + emergency reaction + contributory

negligence of P Consensual cases—bailment analogy

Common law default rule: place loss on cheaper cost avoider D can be held to subjective standard since P chooses D

P takes D as he finds him. Sliding scale of standard of care depending upon whether P or D benefits from

transaction. (Parallels sliding scale in landowner liability)

Have to moderate where to set liability in order to persuade D to take on task. Epstein theory: in consensual situations K should dominate tort.

Guest statutes—owner-driver of car only liable for gross negligence or recklessness to passenger. See also : landowner liability, medical malpractice

Unreasonable risks—what is reasonable care. If D takes reasonable risks, no negligence.

ask if D would have acted differently if all the property threatened and needed to be sacrificed belonged to him

Hand cost-benefit formula: Negligence exists when B (burden) is less than P (probability of harm) times L (injury).

U.S. v Carroll Towing (2nd Cir 1947): Barge broke away, impaled itself on a ship’s rudder, sank. Likelihood of harm and gravity of harm varies with circumstances, so bargee’s duty also depends on circumstances.

Hand formulation leaves cost of accidents unavoidable at reasonable cost on P. Problem: difficult to estimate, especially the injury that might occur; fails to

consider alternatives. When measured ex post, harm always looks foreseeable. RTT: issues that determine whether conduct lacks reasonable care are the foreseeable

likelihood that t will result in harm, the foreseeable severity of the harm that may ensue, and the burden that would be borne by the actor and others if the actor takes precautions that eliminate or reduce the possibility of harm. Blyth v. Birmingham Water Works (Ex 1856): 25 years before accident, public company installed

water pipes with fire plugs. No negligence in construction or laying of pipes. One winter water leaked out of plug and damaged walls of P’s house. Held: reasonable man would have planned for the kinds of weather experience had led him to expect; D’s precautions were sufficient based on that expectation; D did not have to foresee all potentialities.

Osborne v. Montgomery (WI 1931): Bicycle accident. D stopped car, parked in such a way that P bicyclist had to swerve around him, P’s bike hit when D opened car door, causing injury to P. Paris v. Stepney Borough Council (Eng. 1951): D employed one-eyed workman, did not issue

googles. Workman got metal splinter in good eye while working and was blinded. Did D owe duty to this particular workman to provide googles since the harm that could result would be must more severe in its consequences than that to a normal workman? No, P was no more at risk of injury than anyone else.

Emergency and necessity D engages in risky behavior to meet an emergency not of his own making. The risks D takes in order to meet an emergency will be taken into account when

deciding whether D was negligent

21

Page 22: Torts - University of Chicagoblsa.uchicago.edu/first year/TORTS/Torts-Epstein2002b.doc · Web viewTorts Introduction: Three categories of torts Intentional torts: where the defendant

Torts

Exception: if emergency arises from D’s antecedent negligence. Eckert v Long Island RR (NY 1871): cost-benefit analysis goes out the window. Small child on

tracks, man rushed out, throws child off tracks, saving his life but man gets hit by train and dies. Rule: law favors trying to save a life unless act is rash.

Custom —to fill gaps in standards of reasonableness. R2T: In determining whether conduct is negligent, the customs of the community, or

others under like circumstance, are factors to be taken into account, but are not controlling where a reasonable man will not follow them.

Compliance with custom does not conclusively show lack of negligence and lack of compliance does not conclusively show negligence. But, violation of custom creates rebuttable presumption of negligence.

Trimarco v Klein (NY 1982): P injured by glass shower door which was not plate glass as was usual at that point. But building was old. D not liable for customs that have changed since his action.

Compliance with custom : Custom may not be detailed enough to deal with the situation in question. Courts don’t want to let industry use custom as excuse for developing

unreasonably risky practices TJ Hooper (2nd Cr 1932): tug lost in storm, owners of barges lost allege negligence b/c

tug not equipped with radio. Held: fact that industry custom does not require radios does not relieve D of liability. Hand, J.: courts operate as checks to make sure industry does not collude in adopting a

lower than reasonable standard. Mayhew v Sullivan Mining (ME 1884): independent contractor falls down hole in platform

on which he worked. He had not been alerted, nor was hole protected. No custom to enclose hole, but held that P should at least have been warned.

Custom and harms to strangers Custom can’t be decisive since parties to custom have no reason to take strangers

into account. Custom and harms in consensual relationships

View #1: employers are not insurers, test of negligence is ordinary usage of the business—implied terms of employment K set limit of employers liability. Titus v Bradford (PA 1890): D operated narrow gauge RR. It unloaded cars from reg gauge

RR and transferred them onto narrow gauge trucks. This worked if the car bottoms were flat, but Nypano cars were curved. To accommodate, D used wedges bolted or tied in to stabilize the cars. P had worked for D and knew methods to secure Nypano cars. He was riding on a Nypano car. Conductor had visually inspected the wedges and their connections. P’s car started to sway—train going slowly—and P tried to get off it before it fell, but ended up on track and was run over. Problem was that wedges had come loose. Held: no negligence b/c in a dangerous industry, where there is a custom of use, employer can’t nec be shown to be liable even if additional precautions above those used customarily would have been safer and the customary way of securing cars was not shown to be dangerous or unusual in the industry.

View #2: old rule gives firm no incentive to take safety measures b/c it can hide behind custom. Argument is wrong in stranger cases, and in consensual setting higher safety

means lower wages. If D adopts higher internal standard, not held liable at that standard to strangers

But, this view fails to ask if P relied on D’s higher standard b/c D had advertised it, for example.

22

Page 23: Torts - University of Chicagoblsa.uchicago.edu/first year/TORTS/Torts-Epstein2002b.doc · Web viewTorts Introduction: Three categories of torts Intentional torts: where the defendant

Torts

Lucy Webb Hayes National Training School v. Perotti (DC Cir 1969): patient in mental hospital was able to jump out of window b/c the hospital was lax in enforcing its own rules to guarantee patient safety.

Standards reduce problem of trying to run cost-benefit analysis. Customs are contractual proxy: They are chosen ex ante, they are known, they

are designed to prevent accidents, in consensual settings P knows customs are can choose whether to abide by them, K around them, or avoid the relationship.

Medical custom Custom respected in medical malpractice cases: conformity with professional

practice usually held to meet standards of the reasonable physician. Lama v Borras (1st Cir 1994): P has back surgery and ends up with dangerous spinal infection.

Held: care was not to standard of profession. Helling v Carey (WA 1974): outlier. P saw D often b/c of eye irritation. Eventually D tested

her for glaucoma, but which time P suffered permanent damage. Custom in profession not to test for glaucoma in patients under 40. Held: D liable for not giving test regardless of custom. [Result later overturned by statute.]

Problem: medical custom differs. Old locality rule : doctors held to standard of care of other doctors in the area.

Small v Howard (MA 1880): country doctor not held to same level of expertise as big city doctor.

New rule : holds that there is a national standard expected of all doctors. New treatments : Must be followed by “considerable number of physicians” in

order to constitute a legitimate school. Informed consent: what does doctor have to disclose?

Customary disclosures or all material and reasonable disclosures What is material, and material to whom—doctor or patient, which patient

specific or typical? Canterbury v Spence (DC Cir 1972): D did spinal surgery on P, never told P or his mother

how dangerous surgery was. P ended up in worse shape than before surgery. Held: must make all material and reasonable disclosures to enable patient to make informed decision.

Consent requirements: Dr doesn’t have to disclose conditions that are so obvious that they are or

should be known to patient Dr doesn’t have to disclose rare conditions that a reasonable patient would

disregard So, duty to disclose limited to intermediate risk not obvious to patient and

large enough to matter. Cts require expert testimony on what should be disclosed reinstates

importance of industry custom. Statutes:

Three standards adopted by different states: RTT: violation of a statute is negligence per se

Osborne v McMaster (MN 1889): Drug store clerk sells poison without labeling it as Poison in accord with state statute. Buyer takes it and dies. Held strict liability b/c this is public welfare statute. There doesn’t need to be a particular C.L. action for this act b/c violation of a statute is negligence per se.

R3T §12: statutory violations as negligence per se: An actor is negligent is, without excuse, the actor violates a statute that is designed to protect against the

23

Page 24: Torts - University of Chicagoblsa.uchicago.edu/first year/TORTS/Torts-Epstein2002b.doc · Web viewTorts Introduction: Three categories of torts Intentional torts: where the defendant

Torts

type of accident the actor’s conduct causes, and if the accident victim is within the class of persons the statute is designed to protect.

Thayer: once you have a statute, not abiding by it is no longer a negligence question for a jury but negligence per se. Allowing the question to go to a jury would mean allowing individuals to decide what laws are to be followed. Clinckscales v Carver: stop signs not authorized by statute, but still expect them to be

obeyed b/c reliance factor is powerful. Violation of a statute is prima facie evidence of negligence, but can be rebutted by

showing how D’s act is reasonable under the circumstances Tedla v Ellman (NY 1932): statute says you have to walk along road facing oncoming traffice.

P hit by car when walking with back to traffic. Problem: statute codified C.L. about where to walk, but C.L had an exception, that if traffic was lighter on the other side, you should walk there even with your back to traffic. t doesn’t make sense that statute would force pedestrians to do the more dangerous thing when custom was designed to protect them.

R2T & RTT go further allowing greater alternative hazard to displace statute. Violation of a statute is treated as only mere evidence of negligence.

Threshold question: Does the statute create a private right of action that would allow P a statutory cause of action when D does not meet standard of care set out in statute? It’s not always clear if statute creates private cause of action. Common law rule:

person disproportionately injured by statutory breach will be able to sue. Fitzwater v Sunset Empire (OR 1972): D did not clear ice and snow from sidewalk in

accordance with municipal statute. P injured. Held: statute created no private cause of action, no liability to 3rd persons but only a duty to help the municipality keep the sidewalks clear.

P must show: She was a party for whose protection the statute operated—usually the public at

large Her injury was of the type against which statute guards

Gorris v Scott (Ex 1874): animals washed overboard not protected by statute mandating they be shipped in separate pens to avoid contagion, though animals would have been safe if pens had been correctly installed, but statute didn’t cover safety.

Stimpson v Wellington Service (MA 1969): statute prohibiting heavy vehicles from using city streets held to allow P cause of action when D’s truck cause pipe beneath street to burst.

D has no excuse and cannot show P’s contributory negligence There is proximate cause between statutory breach and P’s harm.

Brown v Shyne (NY 1926): D gave chiropractic treatment without a license, resulting in serious injury to P. Held: absence of license was not the cause of the harm.

Ross v. Hartman (DC Cir 1943): D left keys in ignition, T stole car, hit P in accident. Statute against leaving keys in car makes D liable for the harm to P b/c this was what the statute was trying to protect agaisnt.

Courts were reluctant to adopt an absolute negligence per se rule in cases of statutory violation—danger that liability will become strict.

Proving negligence Judge and Jury

Rationales for control of juries Judges fear juries might abuse power by deciding cases against established

principles of law. Judges fear that arbitrary decisions by jury will result in undermining of principle of

distributive justice that like cases should be decided alike.

24

Page 25: Torts - University of Chicagoblsa.uchicago.edu/first year/TORTS/Torts-Epstein2002b.doc · Web viewTorts Introduction: Three categories of torts Intentional torts: where the defendant

Torts

Holmes wanted to standardize verdicts and let judges take more control. Example: cars hit at RR crossings, where there was a single track, clear view, and unmarked crossing—no negligence by D. Baltimore and OH RR v Goodman (US 1927): Goodman killed when train hit his car as it

was crossing a railroad track. Goodman held to have been negligent in going onto the track. Everyone knows tracks are dangerous and extra precautions are necessary, listening and looking from within the car are not enough, have to get out and look down tracks.

But, Pokora v Wabash Ry (1934): P’s car hit as he crossed a series of 4 tracks. He did stop, look and listen, but didn’t get out of car. Found for D b/c P contributorily negligent. Cardozo reversed: “Standards of prudent conduct are declared at times by courts, but they are taken over from the facts of life. To get out of a vehicle and reconnoiter is an uncommon practice, as everyday experience informs us.” In this case, getting out of the care not only might have been dangerous, it wouldn’t have told P whether the tracks would be safe by the time he returned to the car. Goodman has been powerful, though.

Two means of control Instructions to the jury Keeping certain questions of fact from jury Sometimes it does this by determining that a matter does not constitute negligence

as a matter of law. Proving negligence turns on cost of avoidance, probability & severity of harm.

Court has to examine the tradeoffs between increased precautions and their associated benefits. D will look for an alternative hazard that would be created by P’s suggested

precaution. Cooley v Public Service (NH 1940): Power wire falls, hits telephone wire causing loud

noise on line and giving person on the telephone a neurosis. Power company not negligent. If they tried to prevent this problem, they would put people on street at greater risk of electrocution. D has to balance risks: danger to people on street is greater than to people on phone, and D has stronger relationship and therefore duty to people on street.

[Cf. products liability, esp. warning and design defects] D wants trier of fact to consider situation ex ante. P will show how ex post a precaution would have prevented his exact injury.

P will stress precaution’s low cost, efficiency. D will stress its cost, untestedness, inefficiency, creation of alternative dangers,

unreliablility. P can argue, if harmed by object of D’s:

D knew of risk and failed to remove dangerous item D knew of risk but failed to repair it D knew of risk and repaired it poorly D did not know of risk but could have discovered it with better inspections D knew inspections were difficult and should have used a better quality item D should not have used the item at all given its inherent risks.

P has burden to show: What D did How dangerous it was D’s opportunity to discern danger Availability of safer alternatives D’s opportunity to know about safer alternatives

25

Page 26: Torts - University of Chicagoblsa.uchicago.edu/first year/TORTS/Torts-Epstein2002b.doc · Web viewTorts Introduction: Three categories of torts Intentional torts: where the defendant

Torts

Res Ipsa Loquitur Plaintiff was injured by accident that wouldn’t ordinarily happen without

negligence. Plaintiff’s burden of proof is not to eliminate all possible causes of injury but to show

that negligence was the more probable cause. Only need circumstantial evidence

Byrne v Boadle (Ex 1863): barrel falling from warehouse hits P walking by. Negligence can be assumed by the nature of the accident.

Shifts burden of showing no negligence to D. Elements:

The event must be of a kind which ordinarily does not occur in the absence of someone’s negligence,

It must be caused by an agency or instrumentality within the exclusive control of the D, and

It must not have been due to any voluntary action or contribution on the part of P. Exception to the exclusive control standard is in case of non-delegable duties—

allowing delegation would create perverse incentives, because company could escape liability by hiring cheapest, most incompetent subcontractor Colenares Vivas v Sun Alliance Ins Co (1st Cir 1986): Ps injured while riding escalator at

airport when handrail suddenly stopped but stairs kept going. Airport can’t delegate duty of care to travelers, and Ps not doing anything but riding.

Holzhauer v Saks (MD 1997): P injured when escalator stopped abruptly. Ct didn’t allow RIL b/c of possibility that a 3rd party had pushed the emergency stop button, which would defeat 3rd party prong. But, Victory Park Apt v Aexelson (ND 1985): building damaged by fire that started in

P’s apt, due to smoldering cigarettes P’s guests had been smoking earlier. No RIL b/c P not under duty to inspect for smoldering cigarettes left by others.

Exclusive control can allow some plaintiffs to get round worker’s compensation by suing the entity in control.

Possible role of acts of God may defeat res ipsa claim: Walston v Lambersten (ship lost at sea for no known reason): sea is hazardous and inference of negligence cannot be made lightly.

Pursuing res ipsa loquitur claim For whose conduct is D vicariously liable—who is responsible

Larson v St Francis Hotel (Cal App 1948): hotel not responsible when guest threw chair out window on V-J day.

Connolly v Nicollet Hotel (MN 1959): hotel held liable for foreseeable rowdiness of conventioneers.

Chain of custody—third party issues Normal and proper downstream use may obviate need for chattel to be in

actual control of defendant. Benedict v Eppley Hotel (NE 1954): chair collapses under hotel guest who was just

sitting in it. What if liability is not applicable standard of liability

If standard is strict liability or highest standard of care, P wins by showing harm. If standard is lower than negligence, e.g. gross negligence in guest statutes, P can rarely win on res ipsa loquitur. Galbraith v Busch (NY 1935): P, a guest in D’s car, was injured when car

unaccountably swerved off road. Accident could have been caused by negligent

26

Page 27: Torts - University of Chicagoblsa.uchicago.edu/first year/TORTS/Torts-Epstein2002b.doc · Web viewTorts Introduction: Three categories of torts Intentional torts: where the defendant

Torts

driving or negligent maintenance, and NY statute held driver not liable to guest for maintenance and only gross negligence in driving. RIL doesn’t apply to guests.

Pfaffenbach v White Plains: P was passenger in car driven by friend, was injured when truck coming in opp direction skidded across median and hit car. 1) skidding prima facie evidence of negligence, 2) P not a guest of truck driver. RIL can apply when D hits P, a stranger—no fear of collusion, and D owes duty of reasonable care on public streets.

Can negligence be inferred Ybarra v Spangard (CA 1944): P goes into surgery for appendectomy, comes out with

nerve damage in shoulder. Multiple doctors and nurses involved, P doesn’t have to show who was at fault or what happened, b/c P was not contributorily negligent and this was not an act of God scenario. Use RIL to get Ds to tattle on each other.

Greenberg v Michael Reese (Il App 1979): cancer caused by radiation treatment which at the time they didn’t know would cause cancer. No RIL b/c no negligence in the treatment and doctors didn’t know the long-term effects.

Plaintiff’s ConductContributory negligence Definition : conduct on the part of P which falls below the standard to which he should

conform for his own protection, and which is a legally contributing cause co-operating with the negligence of D in bringing about P’s harm. Duty: P has duty to avoid causing harm to himself—minimize harm D could cause

Two approaches: P has no duty to D to take steps to prevent D from harming him: LeRoy Fibre v Chicago, Milwaukee & St Paul Ry (US 1914): P doesn’t have to use his land

such that D’s sparks won’t cause P’s merchandise to catch fire. Seatbelt cases: Derheim v Fiorito Co (WA 1972): P injured in car accident for which she was

in no way responsible, D wants contributory negligence b/c P not wearing seatbelt so injuries more severe. Held: P has no duty to mitigate damages before harm has been caused. D has to take P as he finds her.

Beems v Chicago, Rock Island RR (IA 1882): brakeman killed b/c engineers didn’t slow the train when instructed. P is dependent upon D for his safety, so D is responsible, and P would only be contributorily negligent if he had been reckless.

Alternative view: loss prevention is the responsibility of both parties. Standard of care: P owes standard of reasonable man under the circumstances, where P

assumes D is also taking reasonable care. D has burden of showing P was not taking reasonable care. Gyerman v United States Lines (CA 1972): fishmeal incorrectly stacked. P complained but did

unloading anyway and was injured. P’s options were unclear or limited and D couldn’t prove that it would have remedied the situation. Standard of care of D higher in custodial care situation:

Padula v State (NY 1979): inmates at drug rehab center able to get to and drink copying fluid which has a lot of alcohol in it due to negligence of guards. Ct. allowed suit b/c question of contributory negligence turns on whether injured person judged to have been able to control his actions.

Emergency doctrine and contributory negligence: person faced with an emergency, who doesn’t have time to deliberate, won’t be charged with contributory negligence if he acts as a reasonable person would under the circumstances, even if in hindsight his actions were not the most prudent ones possible. Caveat : no party can rely on an emergency created by his prior negligence

Causation: P’s conduct must have been a substantial factor in bringing about the harm. Berry v Borough of Sugar Notch (PA 1899): tram driver speeding, tree by side of tracks falls in

tram as it passes underneath. Speeding not contributory negligence b/c didn’t change outcome.

27

Page 28: Torts - University of Chicagoblsa.uchicago.edu/first year/TORTS/Torts-Epstein2002b.doc · Web viewTorts Introduction: Three categories of torts Intentional torts: where the defendant

Torts

Smithwick v Hall & Upson Co (CT 1890): P told not to work on east side of a platform along icehouse. D feared P would slip and fall. While P was working on east side of platform, the icehouse collapsed and P was injured. D tried to argue contributory negligence b/c P was working where he shouldn’t have been, but Ct said that P’s conduct was not related to the injury he actually sustained b/c the harm came from a different cause.

Reciprocal causation: Coase, The Problem of Social Cost-- To avoid harm to B would inflict harm on A. Problem is to avoid serious harm in either direction and the answer depends on the value of what is gained and what lost by harming one or the other.

Last clear chance : P is negligent but D had a chance to avoid accident (sequential harms). Applies when D knows, but does nothing about, plaintiff’s predicament, sees P is

helpless or inattentive. Helpless P must show she was unable to prevent harm by exercise of reasonable

care and vigilance Logic: Objective is to induce optimal levels of precaution when action of P and D occur

sequentially and independently and when the second party knows or has reason to know of the risk already created by the first. This reduces likelihood of accidents b/c if contributory negligence bars recovery by P, P will take extra care, but D will have no incentive to take care. This way, if D is aware of risk created by P and his cost of avoidance is low, this doctrine gives D incentive to take precautions to avoid injury to P.

Problem: if one party thinks the other will take care, it will dispense with precautions. Davies v Mann (Ex 1842)=classic case of doctrine: P lets his donkey graze on public highway,

D driving wagon runs down donkey and kills it. Rule is that you have a duty to avoid injury even if the situation is caused by the other party’s negligence.

Fuller v Illinois Central RR (MS 1911): Old man, not paying attention, crossed tracks in wagon. Trainmen saw decedent come onto tracks at such distance that train could have been stopped. Trainmen did nothing except standard whistle blow 20 seconds before crashing into wagon. P claims D had last clear chance to avoid injury.

Where there is a statutory duty, D cannot point to P’s contributory negligence or assumption of risk as valid defenses b/c this would allow D to circumvent statutes in place for protection of Ps. Osborne v Salvation Army (2nd Cir 1939): P injured while cleaning windows, D did not provide

required safety equipment. P’s contributory negligence did not bar claim. Epstein rule on joint liability: divide responsibility equally among parties.

Assumption of risk: D concedes he inflicted harm on P but claims that since she assumed the risk it was as

though P harmed herself. Requires a showing that plaintiff actually knew of risk and chose to proceed.

Express assumption of risk (by K): P expressly agrees not to hold D liable for injury suffered from risk created by D. Requirements

Consent to accept risk is freely given Consent must be to particular risk that led to injury Validity of Disclaimers

Primary (Implied) assumption of risk : fellow servant rule—could not hold employer liable for negligence of fellow employee.

Farwell v Boston & Worchester RR (MA 1842): engineer injured by negligence of another worker, but not allowed to recover from employer under fellow servant rule. Worker took job

28

Page 29: Torts - University of Chicagoblsa.uchicago.edu/first year/TORTS/Torts-Epstein2002b.doc · Web viewTorts Introduction: Three categories of torts Intentional torts: where the defendant

Torts

with knowledge of risks and so if anything happens, it is an accident and he has no remedy against employer.

Proposition that D was not negligent, i.e. either owed no duty or did not breach the duty owed. Specific knowledge of particular hazard, not participation in a dangerous

occupation is vital to assumption of risk defense. Lamson v American Axe and Tool Co (MA 1900): P painted hachets for company.

Once painted he placed them on a rack. D replaced racks, and P complained that hatchets fell off of new racks, creating risk of injury to him. D said, live with it or quit. P continued to work, and was injured by a falling hatchet. Holmes, J., worker knew the likelihood of injury, but he chose to continue taking the risk. Although the injury did not depend upon negligence, he assumed risk and therefore has no suit.

Murphy v Steeplechase Amusement (NY 1929): P went on ride at amusement park where people got knocked off their feet. Sides and floors were padded. P knew what went on. P was injured. Held: it was clear what ride involved, there were no special departures from normal workings. If you take part in sport where the danger is known, obvious and inherent and you agree to it, you assume the risk. It would make a difference if the dangers had been boscured, unobserved, or so serious as to justify belief that precautions should be taken.

Spectator sports and assumption of risk: Uniformly held that you assume a certain risk when you are a spectator at a sports event.

1. common knowledge of the possibility of injury2. evidence usually is that specific P’s knew of risk of injury

Professional athlete assumes risk when he agrees to play, especially when he appreciates the existence of a particular danger: Maddox v City of NY (NY 1985): baseball player injured due to muddy outfield about

which he had complained to grounds keepers. He knew of the risk and potential for injury.

Secondary assumption of risk : whether the reasonably prudent man in the exercise of due care (a) would have incurred the known risk, and (b) if he would, whether such a person in the light of all the circumstances including the appreciated risk would have conducted himself in the manner in which P acted.

Meistrich v Casino Arena Attractions (NJ 1959): P injured while skating even though he chose to skate after learning that the ice had been improperly prepared. So P carelessly contributed to his injury.

Marshal v Ranne (TX 1974): P bit by neighbor’s wild boar as he walked from house to car. P knew boar was here and had had chance to shoot it but hadn’t done so. Trial Ct denied recovery b/c P had been contributory negligence in not shooting the boar. Supreme Ct reverses b/c P didn’t have a choice, either he could stay in house or he could risk being bitten by boar, and he had a right to leave the house.

Letting Ps get away with assuming risk b/c D gave insufficient warning of danger is an implicit repeal of the assumption of risk doctrine.

29

Page 30: Torts - University of Chicagoblsa.uchicago.edu/first year/TORTS/Torts-Epstein2002b.doc · Web viewTorts Introduction: Three categories of torts Intentional torts: where the defendant

Torts

Comparative NegligenceTraditional contributory negligence barred all recovery by P.

Pure comparative negligence regime simply limits P’s recovery in proportion to his fault.A. Comparative negligence attempts to divide liability between P and D in proportion to their

relative degrees of fault.a. P’s fault is not relevant if it is not one of the causes in fact or proximate causes of

P’s harm.b. In negligence cases compare P’s fault with D’s fault, don’t compare causation.

i. Each party is liable for damages in proportion to his share of the fault.1. Li v Yellow Cab (CA 1975): D is speeding while P is illegally changing lanes.

Established comparative negligence in CA.ii. In strict liability, the apportionment is of comparative causation.

1. Bohan v Ritzo (NH 1996): D’s dog ran in menacing way at P riding bike. As P tried to ward off dog, he fell and broke his arm. Dog never touched P. Held: statute makes control of dog a strict liability issue. There is no fault in SL, but Ct interprets comparative fault in SL cases as comparative causation. Here, P did nothing to cause attack, so he can recover all damages.

B. Pure vs. 50% systemsa. Pure comparative negligence regime apportions the loss in accordance with the

relative fault of the parties (adopted in 13 states) b. Modified comparative negligence regimes, the traditional bar to recovery remains

until P’s fault reaches a specified percent.i. Greater-than version: P is barred if his fault exceeds that of D (adopted in 9

states)ii. Equal to version: P is barred unless her fault is at least equal to that of D

(adopted in 21 states)1. Problem: most cases fall around 50-50 split, so this is inefficient and

juries will work around it anyway in order to award damages to P2. But you get fast settlement b/c everyone knows the trial will be a

crap shoot.c. FELA: contributory negligence doesn’t bar recovery. No comparative negligence where employee

not guilty of contributory negligence where common carrier’s violation was of a statute enacted for safety of employees.

C. Considerations:a. Fairness versus efficiency

i. You want to give appearance of justice so people believe the law is fair, butii. You want to encourage swift resolution of cases, especially small ones.

1. If you indulge in fairness you hurt efficiency b/c it makes the system difficult to administer since no one knows how to measure proportions of fault.

2. Individualized justice is too costly in cases where the negligence of each party is very similar

b. Epstein rule: divide damages by number of parties and everyone gets a pro rata share.

i. Simple to administer, will encourage settlement, and most cases will fall around 50-50 division.

c. Use special verdicts to control juries in comparative negligence cases in order to unpack severity from culpability and aid judicial review.

30

Page 31: Torts - University of Chicagoblsa.uchicago.edu/first year/TORTS/Torts-Epstein2002b.doc · Web viewTorts Introduction: Three categories of torts Intentional torts: where the defendant

Torts

D. Comparative negligence & …1. Last clear chance : unnecessary under comp negl regime.2. Assumption of risk : where there is primary assumption of risk (P knows of risk but goes ahead

anyway), D is not at fault, so comp negl is not at issue. Where there is secondary assumption of risk (D has breached duty of care to P but P, knowing this, goes ahead anyway), there can be comparative negligence, so no need for an assumption of risk defense that shifts whole burden to P.

3. Wilfull misconduct : Cts disagree. Burke v 12 Rothschild’s Liquor Mart (Ill 1992): negligence and willful misconduct are

qualitatively different and can’t be compared. Sorenson v Allred (Cal App 1980): P’s recovery diminished by her negligence even though

D’s misconduct was willful and wanton.4. Intentional torts : Cts differ.

Munoz v Olin (Cal App 1977): wrongful death action against policeman who shot decedent suspected of arson. Comparative negligence is not a substitute for the defense of privilege to commit an intentional tort. If P’s action made it seem necessary to D to act as he did, that is sufficient to remove liability from D.

Blazovic v Andrich (NJ 1991): intentional conduct is not different in kind from negligence and willful misconduct, but different in degree b/c it requires knowledge rather than possibility of harm. Jury’s apportionment of fault will reflect different levels of culpability of each type of tort.

5. Violation of a safety act : Under comparative negligence, an employee’s violation of a safety act will result in his being responsible for some of the harm whereas before the employee’s contributory negligence was not a defense for D. This is designed to give employee incentive to act safely.

But FELA makes exception for violation of safety statutes by employer.6. Avoidable consequences : Ostrowski v Azzara (NJ 1988): P sued D for malpractice resultant from

complications sustained after routine surgery. D argues complications could have been avoided if P had taken the precautions D suggested. Comparative negl held to apply where P fails to mitigate damages.

7. Seat belt defense : by statute often remains good law regardless of adoption of comparative negligence8. Imputed negligence : LaBier v Pelletier (ME 1995): Mother was not watching her child who ran into

street and was hit by D’s car. Trial ct instructed jury to find for P if combined causative negligence of parent and child was greater than that of D. Appeals Ct remands. R2T §488: “a child who suffers physical harm is not barred from recovery by the negligence of his parent, either in the parent’s custody of the child or otherwise.”

9. Joint and several liability : the adoption of comparative negligence does not by itself eliminate the rule of joint and several liability or the rules governing the redistribution of damages among the Ds.

American Motorcycle Association v Superior CourtE. Multiple Parties: comparative negligence runs into problems when P joins two or more Ds

in a suit.a. Example 1: P, D1, D2 are all 1/3 liable.

i. Pure comparative negligence: P is 1/3 liable and recovers 1/3 from each Dii. Modified regimes:

1. treat it as two suits: P v D1 and P v D2, where in each case P and D are 50% liable

a. Under equal to regime, P recovers 50% from each Db. Under greater than regime, P recovers nothing

2. Measure P’s negligence against the combined negligence of the Dsa. P recovers 1/3 from each D

31

Page 32: Torts - University of Chicagoblsa.uchicago.edu/first year/TORTS/Torts-Epstein2002b.doc · Web viewTorts Introduction: Three categories of torts Intentional torts: where the defendant

Torts

b. When 1 D is insolvent who pays his share?i. Under common law, risk of D1’s insolvency fell on D2, as long as P was not

negligent and thus barred recovery.ii. Apportionment treats D1 and D2 as responsible for separate harms as though

they were separate tortfeasors.1. So if D1 were insolvent, P can only recover the 1/3 share from D2.

iii. Another position sticks to common law rule on the ground that the move to comparative negligence was intended to increase P’s compensation

1. P would continue to recover the full 2/3 from the solvent Div. Intermediate position apportions the insolvent’s share between D2 and P

1. E.g. P = 10%, D1 = 60%, D2 = 30 % negligent.a. P recovers the 30% from solvent D2 plus insolvent D2’s 60%

share is divided between P and D1 on a 10 to 30 (1 to 3) ratio, so P also recovers 45% of D1’s share from D2.

Multiple PartiesA. Joint & Several Liability Joint liability : implies that each of several Ds is responsible for the entire loss that they all

caused in part. Several liability : holds each D responsible only for his proportionate share of the loss.

1. Common law background: hostility toward apportionment in contributory negligence cases, so P could decide which D would bear all costs.

a. Only option for D: to seek an indemnity from codefendant for the full amount of the loss.

1. Merryweather v Nixan (KB 1799): P sued two Ds for conversion. Ruling: if P recovers all of damages against one D, that D cannot recover a part from the other.

2. Union Stock Yards v Chicago, Burlington & Quincy RR (US 1905): Faulty nut in switching car caused injury to P’s employee. Either P or D could easily have found and rectified problem. Employee recovers full amount from P who wants contribution from D. Held: usual rule = one D cannot recover against another D, but exception is when one D is less culpable than the other b/c the second D is principally responsible for the damage. E.g. Gray v Boston Gas & Light (MA 1873): D attached telegraph cable to P’s chimney w/o P’s permission. Cable pulled chimney down and it injured passerby on the street. P can recover against D b/c parties were not in pari delicto. But in this case, the two companies were guilty of like neglect of duty, so exception doesn’t apply.

2. Release of joint tortfeasors:a. Common law rule: if P released one of several tortfeasors, he lost suit against all

others.b. Recent trend is to reject automatic release at least where tortfeasors are

independent of one another, e.g. driver who causes injury and doctor who treats victim and commits malpractice.

3. Joint and several liability under comparative negligence where apportionment is not possible:

a. Retain joint and several liability as a way to increase P’s recoveryi. American Motorcycle Association v Superior Court (CA 1978): P, a minor, injured in

motorcycle race organized by AMA. AMA sought to file cross-complaint against P’s

32

Page 33: Torts - University of Chicagoblsa.uchicago.edu/first year/TORTS/Torts-Epstein2002b.doc · Web viewTorts Introduction: Three categories of torts Intentional torts: where the defendant

Torts

parents for improper supervision of their minor son, and asked that its portion of judgment be reduced by the amount of the allocable negligence of the parents. Held: (a) the simple feasibility of apportioning fault on a comparative negligence basis does not render an indivisible injury ‘divisible’ for purposes of the joint and several liability rule. The mere relative culpability of one negligent D as compared to another does not in any way suggest that each D’s negligence is not a proximate cause of the entire indivisible injury. (b) A P may be completely free of liability, but if you abolish the joint and several doctrine, the P will have to bear burden of damages if one of the Ds can’t pay his proportionate share.

b. Use only several liability in which each D is responsible only for his share of the total loss, usually defined in terms of causal negligence.

4. Alternative liability: 3 scenariosa. Act of each D is the necessary cause of P’s harm, without Ds’ acts, there would have been no

harmb. Act of each D is a sufficient cause of the harm, such that neither act is a necessary onec. Act of either D1 or D2, but not both, caused the harm and evidence is insufficient to determine

which one caused the harm.i. Summers v Tice (CA 1948) D1 and D2 each fired a bullet at the same time, one bullet

hit P, and it is not clear from which gun it came. Both Ds held jointly and severally liable on theory of alternative liability.

5. Adjustment of liability among D after some or all damages have been paid to P. (In re Amoco Cadiz, 7th Cir 1992)

a. No contribution: all Ds are jointly and severally liable for the full damages. P may collect from any of them, but they cannot get contributions from each other.

b. Contribution: all Ds are jointly and severally liable for the full damages. P may collect from any of them, a party asked to pay more than his share can recover the excess from a party asked to pay less than his share.

c. Contribution plus settlement bar: all Ds are jointly and severally liable for the full damages. P may collect from any of them, but one D may obtain contribution only from another D that proceeds to judgment. A settling party escapes liability for contribution.

d. Claim reduction: Ds are jointly and severally liable, unless one of more settles. By accepting a settlement from one party, P foregoes the ability to collect from the remaining Ds any damages attributable to the settling party’s share of the fault. The remaining Ds are not entitled to contribution from the settling party. (Comparative fault rule.)

i. Once each D’s liability is determined, they are treated individually in relation to P.

ii. Reduces externalities

C. B. Vicarious Liability1. Vicarious liability: liability for the tort of another person

o Vicarious liability is not personal liability. It is not based upon the D’s own fault, but only in situations in which he is held liable for the wrongs of another.

2. Respondeat superior : employers are jointly and severally liable along with the tortfeasor employee for the torts of employees committed within the scope of employment.

a. Applies to all torts: The doctrine applies to all torts, including intentional ones and those in which strict liability exists, provided that the tort occurred during the scope of the employee’s employment

33

Page 34: Torts - University of Chicagoblsa.uchicago.edu/first year/TORTS/Torts-Epstein2002b.doc · Web viewTorts Introduction: Three categories of torts Intentional torts: where the defendant

Torts

b. From employer’s perspective vicarious liability is strict liability since employer is liable without personal fault

c. In theory, employer has right of indemnity against tortfeasor employee, but this is rarely exercised.

3. Employer is liable even if the employee acts in a way the employer had expressly forbidden.

4. Scope of employmenta. Employee has to be acting under employer’s controlb. Servant’s conduct is within the scope of his employment if it is of the same general

kind as authorized or expected, or incidental to such conduct, andc. The Servant was acting within the authorized time and space limits.

i. Master is still vicariously liable if servant acts outside scope of employment if employment aided him in accomplishing the tort.

d. Bright line rule: if you are on work time & on the premises of the worksite, you are under the liability of the employer.

e. Ira S. Bushey & Sons v. United States (2d Cir. 1968): Drunk sailor who lives on boat in drydock, damages boat and dock. Held that there is vicarious liability b/c action was foreseeable and related to sailor’s duties.

f. Most courts hold that where an accident occurs where the employee is travelling from her home to work, she is not acting within the scope of her employment. If the employee is returning home after business, courts are divided.

g. Frolic and detour : Even a detour or side-trip for personal purposes by an employee may be found within the scope of employment if the deviation was reasonably foreseeable.

i. Deviation had to be slight, or is usual, expected, toleratedii. Employee is acting in part in service of employer

h. Even if the act done was expressly forbidden by the employer, it will be "within the scope of employment" if done in furtherance of the employment.

i. An intentional tort does not relieve the employer of liability if it relates to the employment.

i. But if the employee merely acts from personal motives, the employer will generally not be liable.

Intentional torts: where supervisor’s act is not directly related to his duties, but P can prove that employer was negligent in not stopping the conduct b/c it knew or should have known it was going on, this makes employer liable.Sexual harassment is not conduct within the scope of employment, but employers are vicariously liable for the creation of a hostile environment by a superior with direct or successive higher authority over victim. Affirmative defenses available to employer:

1. employer that takes reasonable care to prevent and promptly correct any harassment2. employee who unreasonably failed to take advantage of preventive or corrective

opportunities provided by employer5. Employees versus independent contractors

d. An employee is one who works subject to the close control of the person who has hired him.

e. An independent contractor, by contrast, although hired to produce a certain result, is not subject to the close control of the person doing the hiring.

34

Page 35: Torts - University of Chicagoblsa.uchicago.edu/first year/TORTS/Torts-Epstein2002b.doc · Web viewTorts Introduction: Three categories of torts Intentional torts: where the defendant

Torts

f. The "control" required to make a person an employee rather than an independent contractor is usually held to be control over the physical details of the work, not just the general manner in which the work is turned out.

6. One who hires an independent contractor is not generally liable for the torts of that person.a. Exceptions:

i. First, if the employer is herself negligent in her own dealings with the independent contractor, this can give rise to employer liability.

ii. Second, there are some duties of care that are deemed so important that the person doing them will not be allowed to delegate them to anyone.

1. Duty of lateral support, can’t be delegated, blasting or cutting electrical conduits, water pipes or gasoline lines below ground.

iii. Finally, one who employs an independent contractor will also be liable where the work is such that, unless special precautions are taken, there will be a high degree of danger to others.

1. R2T §427: Negligence as to danger inherent in the work : One who employs an independent contractor to do work involving a special danger to others which the employer knows or has reason to know to be inherent in or normal to the work, or which he contemplates or has reason to contemplate when making the K, is subject to liability for physical harm caused to such others by the contractor’s failure to take reasonable precautions against the danger.

2. This special rule of vicarious liability applies only to "peculiar risks," i.e., risks differing from commonly-encountered risks.

3. Employer only liable if the independent contractor is negligent.Borrowed servant doctrine—when an employer hires a sub-contracting employee:

Traditional rule: as long as the servant is furthering the business of his general employer by serving another, there is no inference of a new relationship unless command has been surrendered, and surrender is not to be inferred from mere fact of its division

Modern rule: holds both the general and the special employer liable7. Justification for vicarious liability:

Deep pockets—loss-spreading, employer can better pay Efficiency—employer is superior risk bearer b/c of access to insurance, reduces

chance of an uncompensated risk to P, and P knows someone in the company will be responsible for damage without having to know who.

Employer can go after employee where appropriate Gives employer incentive to hire best, most competent employees

Joint enterprise: rule that holds each partner to a joint enterprise vicariously liable for the wrongs of another partner.

b. Requirements for joint enterprise: There are four requirements for a joint enterprise: i. an agreement, express or implied, between the members;

ii. a common purpose to be carried out by the members; iii. a common pecuniary interest in that purpose; and an equal right to a voice in the enterprise,

i.e., an equal right of control.

35

Page 36: Torts - University of Chicagoblsa.uchicago.edu/first year/TORTS/Torts-Epstein2002b.doc · Web viewTorts Introduction: Three categories of torts Intentional torts: where the defendant

Torts

8. Vicarious liability and medical malpractice:Epstein wants to solve this by making it a contract problem, not a tort issue. The parties are in privity, they can negotiate and distribute risks and costs.Otherwise apply business judgment rule to allow HMO’s to make judgments in good faith and be free from corporate liability.Petrovich v Share Health Plan (IL 1999): Decedent sued HMO and physician for malpractice. HMO claims it can’t be held vicariously liable b/c the doctors with which it contracted were independent operators. Ct. allows liability to go to jury on doctrines of apparent and implied authority.Apparent Authority: a principal will be bound not only by the authority that it actually gives to another but also by the authority that it appears to give. Functions like estoppel. Where the principal creates the appearance of authority, court won’t allow it to deny agency.

Elements to prove: Holding out by principal—giving appearance of authority Justifiable reliance by P on that apparent authority consistent with ordinary care and

prudenceo Where person has no choice but to enroll with HMO, the person justifiably

relies on HMO to provide health careImplied Authority: whether the alleged agent retains the right to control the manner of doing the work. Where a person’s status as independent contractor is negated, liability may result under doctrine of respondeat superior.

When HMO controls doctor’s exercise of medical judgment and that control is negligent, the HMO is liable.

Causation(In a claim based upon negligence)

A. Cause in Fact —did D’s negligent conduct cause P’s harm.B. Proximate Cause —the appropriate scope of D’s legal responsibility for negligence that

has in fact caused harm. (Not really about cause at all, but about consequences—we know what caused the harm, we want to know if D is liable.)

D. Cause in Fact: the cause that, as a factual matter, contributed to P’s harm.But for test: D’s conduct is a cause of the event if the event would not have occurred but for

that conduct; conversely, D’s conduct is not a cause of the event if the event would have occurred without it.

If you can take the conduct out of the scenario, and the harm wouldn’t have happened, the conduct was a cause in fact.

Conversely, if the injury would still have happened, the conduct is not a cause in fact.Multiple “but for” causes

D’s act does not have to be the sole ‘but for’ causeIt is no defense for one negligent actor that someone else’s negligence also contributed

to the harm.The result would be joint liability.

the negligent acts could take place sequentially or simultaneously

36

Page 37: Torts - University of Chicagoblsa.uchicago.edu/first year/TORTS/Torts-Epstein2002b.doc · Web viewTorts Introduction: Three categories of torts Intentional torts: where the defendant

Torts

R2T §433A: Apportionment of harm to causes with multiple tortfeasors(1) Damages for harm are to be apportioned among two or more causes where:

a. There are distinct harms, orb. There is a reasonable basis for determining the contribution of each cause to

a single harm.(2) Damages for any other harm cannot be apportioned among two or more causes.

Comment: Such an apportionment is commonly made in cases of private nuisance, where pollution from different sources has interefered with P’s use or enjoyment of his land. Thus where 2 or more factory owners have polluted a stream, their interference with P is divisible in terms of degree and can be apportioned based of the amount of pollution they each discharged.

Smith v JC Penny (OR 1974): P’s coat made of flammable material, coat bought at Penny’s set on fire by gas station attendant. Injuries determined to be indivisible.Maddux v Donaldson (MI 1961): P’s car hit by X’s car, then hit by Y’s car. Issue: whether successive acts of negligence can be treated as a joint tort in the absence of concerted action by Ds. Divided ct treated it as one tort (which might make one D liable for more damages than his own harm caused), though other authorities require P to separate the harms.

Applying the ‘but for’ test requires deciding whether D’s negligence contributed to the harm or whether it was irrelevant to it. Consider how the event would have worked out if the negligence had not been present. If the harm would have occurred in any case, D is not liable, even if he was in another way negligent.NY Central RR v Grimstad (2nd Cir 1920): barge captain knocked off barge, wife fails to save him.

Can’t show that proper safety gear would have change the result.Burden of proof shifted to D to show that his negligence did not cause the harm

Haft v Lone Palm Hotel (CA 1970): somehow father and son drowned in pool. Hotel violated statute b/c had neither lifeguard no sign posted that there was no lifeguard. Don’t require Ps to establish causation to a great certainty, but rather place burden on D who had violated statute in the first place, to show that it’s negligence was not the cause.

Problem scenarios in cause in fact—exceptions to the use of the ‘but for’ test:Where two Ds act negligently and either one’s act would have sufficed to have caused

P’s injury. (Fires)a. Apply substantial factor testb. Kingston v Chicago & NW Ry—two fires join and burn down P’s house.

Where two or more D’s simultaneously commit the same act, one of which caused P’s injury, but P cannot ascertain which one. (bullets)

Ds share damages pro rataBurden of proof shifts to each D to show that he did not cause the harm—focal

point liabilitySummers v Tice (CA 1948)—two shooters, one of whose pellets hits P

A has 50% probability of doing harm, and B has 50% probability of doing harm (drugs/toxicity)

R2T §433B(3): Alternative LiabilityWhere the conduct of two or more actors is tortious, and it is proved that harm has been caused to the P

only by one of them, but there is uncertainty as to which one has caused it, the burden is upon each such actor to prove that he has not caused the harm. The theory of alternative liability dictates that tortfeasors who act in concert will be held jointly and severally liable for the P’s injury unless the tortfeasors are able to prove that they have not caused the harm.

Four forms of cause in fact issuesScientific connection—scientific/medical doubt about whether one thing caused

anotherZuchowicz v United States (2nd Cir 1998): P had prescription filled at Naval hospital. Prescription

erroneously told her to take twice the maximum dosage. She immediately started having bad

37

Page 38: Torts - University of Chicagoblsa.uchicago.edu/first year/TORTS/Torts-Epstein2002b.doc · Web viewTorts Introduction: Three categories of torts Intentional torts: where the defendant

Torts

side effects, ended up being diagnosed with primary pulmonary hypertension and dying. Prior to taking the drug, she was healthy. Expert testified that the drug side effects were same as those of PPH and that he thought the overdose caused the PPH. Held: If (a) D was deemed to have acted wrongfully b/c the act increased the chances that a particular type of accident would occur, and (b) a mishap of that very sort did occur, this is enough to support a finding that the negligence caused the harm. The D then has to show evidence denying the but for cause.

•In medical malpractice, there is no strict liability, so causation is more complex

Products liability with an unknown cause of harm—P was injured by someone/ something but can’t identify the exact cause:

Engberg v Ford Motor Co (SD 1973): battle of the experts about whether seatbelt worked properly, left to jury b/c D couldn’t show P’s claims were controverted by physcal facts.

General Electric v Joiner (US 1997): P worked I proximity to dielectric fluids that turned out to have PCBs and dioxins in them. P got lung cancer—he had been a smoker, but argued that the PCB exposure hastened the onset of cancer. P’s experts alluded to inconclusive research. Trial judge disallowed their evidence, saying it was too tenuous. Issue: whether judge can decide to admit expert testimony. Held: judge is the gatekeeper to ensure that scientific testimony is both relevant and reliable (Daubert standard).

Daubert v Merrell Dow (9th Cir 1995): P assembled recognized experts to testify that a drug caused birth defects by reinterpreting research that said it didn’t. Daubert overturned Frye v United States (DC Cir 1923) standard which allowed as admissible only expert testimony generally accepted as reliable by the scientific community. Said focus should not be on general acceptance but on reliability and relevance.--Lowered standard about who can be an expert, but can’t be just anyone: court has to ask if the

expert is qualified. An unclear standard.Kumho Tire Co v Carmichael (US 1999) expanded gatekeeper role to technical evidence.

Levels of causation that must be established in toxic torts:1. substance source—substance for which D is responsible caused his harm2. exposure causation—he was in fact exposed to the substance is a way that caused the disease (often a

signature disease)

Market share liability scenario. [Established with DES cases in Sindell v Abbott Labs (CA 1980)]Criteria:

all named Ds are potential tortfeasorsthe products are identical and share the same defective qualitiesP is unable to identify the exact D who caused her injury through no fault of

her ownsubstantially all of the manufacturers who created the product at the relevant

time are named Ds.Skipworth v Lead Industries Assoc (PA 1997): P sued all manufacturers of lead paint for lead

poisoning. P can’t identify whose paint it was, when it was applied in the house, etc. P sued on theory of market share liability, which court refused to apply in products liability action. But PA requires P to identify exact D. Market share liability is a bad idea here b/c (1) the possible time period is too long (100 years), (2) it is impossible to determine when the poison paint was applied, (3) lead paint is not fungible. Thus MSL would distort liability.

Would safe behavior have avoided injury? –D acted negligently, but it can’t be shown that if he had acted in a safe manner he would have avoided causing the harm. D’s negligence was not a cause in fact and he is not liable.

38

Page 39: Torts - University of Chicagoblsa.uchicago.edu/first year/TORTS/Torts-Epstein2002b.doc · Web viewTorts Introduction: Three categories of torts Intentional torts: where the defendant

Torts

Reynolds v Texas & Pacific Ry (LA 1885): P slipped on stairs in unlit hallway. Where harm could have occurred anyway, but D’s negligence made it more probable, the simple fact that it could have occurred anyway doesn’t break causal chain.

Fedorczyk v Caribbean Cruise Lines (3rd Cir 1996): P fell in tub, claimed the abrasive strips were too far apart. Held: P did not present evidence of direct causation.

What harm was caused by D’s negligent conduct?Herskovits v Group Health Cooperative (WA 1983): D negligent in not diagnosing P’s cancer on first visit; b/c of this, P’s chance of short-term survival decreased 14%, but P still would have had less than 50% chance of survival anyway. Issue: is such a decrease in chance of survival enough to go to a jury. Held: yes, P doesn’t have to show there was better than a 51% chance if D had not been negligent. Concurrence: Causation is not an all or nothing proposition. What caused the loss is a separate question from what the nature and extent of the loss are. Allow recovery for the loss of the chance of cure even though the chance was not better than even. Dissent: allowing a lower standard would be too loose. P in a malpractice case must prove that D’s negligence, in probability, caused the death.

[Court got the numbers all wrong. If D had 1% chance of survival and early detection would have made it 2%, do we really want to give that fractional recovery?]

E. Proximate Cause: the appropriate scope of liability for harm.E.g. Central of Georgia Ry v Price (GA 1898): RR passes station where P is to get off, takes her to next stop,

gets her a hotel room, where she is injured. RR held not liable b/c though its negligence put her in the hotel, it was the hotel’s negligence that caused her injury.

E.g. Brower v NY Central & HRR (NJ 1918): collision case, people stole P’s goods after collision and RR detectives did nothing to stop them. Held that the collision resulted in the thefts since it deprived P of the ability to protect his own property. Dissent: don’t confound proximate cause with opportunity for crime.

Goal of proximate cause requirement is to limit D’s liability to the kinds of harms he risked by his negligent conduct.

The issue of proximate cause does not arise until negligence and causation in fact have been proven.

Ex: surgeon negligently performs vasectomy; patient fathers child; child at age 13 sets fire to barn. Surgeon is but for cause, and is negligent, but is not proximate cause.

There can be multiple wrongdoers who are proximate causes of the harm.If 2nd wrongdoer’s conduct intervenes after D’s act is complete. This new cause

would be a superseding cause, and D would not be liable.Dependent causation: each of 2 successive acts is sufficient to harm P, but P is

exposed to the 2nd cause only b/c of the prior negligence of the 1st. The 2nd cause is dependent on the first, so that 2nd D is usually liable only for the additional damages caused by his action.

Basis of proximate cause:1. Either, it is necessary to show at the outset that there was some minimum probability

that the harm would occur given the negligence or other wrongful conduct of the D.2. Or, it is sufficient to trace back causation through intervening events, to establish

liability.Foreseeability test—whether D, at the time he acted, could have foreseen the risk that

injured the P.Must consider what the risks were that made D’s conduct negligent in the first place.Problem: harm is freakish and unforeseeable result, yet D is clearly liable.

39

Page 40: Torts - University of Chicagoblsa.uchicago.edu/first year/TORTS/Torts-Epstein2002b.doc · Web viewTorts Introduction: Three categories of torts Intentional torts: where the defendant

Torts

Doughty v Turner Mfg Co (QB 1964): cover of vat knocked into vat of molten liquid. No one was hurt by splashdirect result. But then chemical reaction with cover caused explosion. Court didn’t let P argue he was harmed by D’s negligence in knocking cover into vat b/c he wasn’t injured by the foreseeable risk.

Problem: if some results were foreseeable, shouldn’t D be liable for unforeseeable resultsPetition of Kinsman Transit Co (2nd Cir 1964): ship breaks away from mooring during winter,

crashed into another ship and breaks its moorings. The two ships run into a drawbridge, ice flows back up, creating dam and miles of land are flooded. “We would find it difficult to understand why one who had failed to use the care required to protect others in the light of expectable forces should be exonerated when the very risks hat rendered his conduct negligent produced other and more serious consequences to such persons than were fairly foreseeable when he fell short of what the law demanded.” Concern that liability has to be limited to foreseeable harms conflicts with thin skill rule, where you don’t know how you will find P and you are liable for all consequences of the harm.

Thin skull rule-where an injury is foreseeable, if you cause a more serious harm, you are still liable

You take your victim as you find himOverseas Tankship Ltd v Morts Dock (Wagon Mound No. 1) (Aust 1961): D negligently discharged oil

from their ship while in Sydney Harbor. The oil flowed to P’s wharf. P, seeing it, stopped his men’s work and inquired whether oil was flammable. Told no, allowed his men to go back to work but told them to be careful. Oil ignited when welding sparks from wharf caught on cotton that was trapped in oil, destroyed wharf.

Direct Cause Test—ex post: trace the harm back looking for breaks in the chain of causation.In re Polemis & Furnis, Withy & Co. (KB 1921): Appellants chartered a vessel from respondents to carry

cargo including gas. While unloading, plank fell into hold, gas exploded, destroying ship. Falling plank found to be cause of the fire. Falling plank was negligence, so appellant is liable for the consequences of the negligence, and since plank caused fire, for the fire.

Polemis rule: what one can foresee is important and may be decisive in determining negligence, but it is not decisive in determining proximate cause. If act is negligent—b/c person had some reason to be able to anticipate it might happen—then guilty party is liable for all the proximate consequences of the act, foreseeable or not. Natural and proximate = consequences which follow in unbroken sequence, without an intervening efficient cause, from the original negligent act.

Increase in risk/restabilization of the situation test/duty: D has to have duty of care for P. If there is no duty, there is no liability.As long as P has not returned to his ex ante stable position, the consequences of D’s act

are still within proximate cause.If D’s act are still causing an increased risk or hazard, the consequences are within

proximate causeMarshall v Nugent (1st Cir 1955): The problem with deciding when the forces set in motion by the harm come to rest:

Union Pump v Allbritton (TX 1995): Pump caught on fire and ignited surrounding area. This pump had caught on fire before. Allbritton, an employee of Texaco, helped put out the fire as she was directed. In following her supervisor to check out a leaky valve on the pipe, P took a dangerous route, slipped and was injured. P sues Pump manufacturer saying that the defective pump led to her injuries, as the but for cause. Issue: where to draw the line on causation. Legal cause is not established if the D’s conduct or product does not more than furnish the condition that makes the P’s injury possible. By the time P slipped, the forces generated by the fire had come to rest. Dissent: in fact the forces had not come to rest b/c the whole place was still covered in water and foam and P was still in fire gear. Also, there was no negligent 3rd party involved inflicting the injury.

40

Page 41: Torts - University of Chicagoblsa.uchicago.edu/first year/TORTS/Torts-Epstein2002b.doc · Web viewTorts Introduction: Three categories of torts Intentional torts: where the defendant

Torts

Complication One—when does chain of causation break?P acts in real or perceived emergencyRule in emergency cases: That if D by negligence puts the P under a reasonable

apprehension of personal physical injury, a right of action arises to recover for the physical injury and the mental disorder naturally incident to its occurrence.a. City of Lincoln (1889)—ship damaged in collision in which it lost its navigational tools, so

captain could not bring it to port. Held that the collision, not the captain’s actions were the cause of the loss of the ship.

b. Jones v Boyce (KB 1816)—P jumped from D’s coach after D had negligently lost control of it. P broke his leg. P’s jump not the cause, but rather D’s negligence. P excused b/c his reaction was a reasonable emergency response to the negligence.

Intentional harm of a 3rd party that wouldn’t have occurred without the but for causeThe P had not yet been returned to a position of stability

Hines v Garrett (VA 1921): RR drops P off 1 mile past her stop in desolate area at night. Walking back, she is raped twice. RR held liable b/c its negligence created the dangerous situation, and the intentional harm of a 3rd party doesn’t sever the causal connection.

R2T §448: Intentionally tortious or criminal acts done under opportunity afforded by actor’s negligence.

The act of a 3rd person in committing an intentional tort or crime is a superseding cause of harm to another resulting therefrom, although the actor’s negligent conduct created a situation which afforded an opportunity to the 3rd person to commit such a tort or crime, unless the actor at the time of his negligent conduct realized or should have realized the likelihood that such a situation might be created, and that a 3rd person might avail himself of the opportunity to commit such a tort or crime.

Last wrongdoer ruleTraditionally, D was held liable only when he was the last wrongdoer whose conduct contributed to the

loss. The last wrongdoer need not be the last actor, just the last blameworthy actor. The test blocks recovery both when the deliberate wrong of a 3rd party intervenes and when the negligence of a 3rd party intervenes.

Modern: negligence of a third party did not sever the causal connection to a prior actor.

Example: P injured in car crash, injured again when ambulance crashes. Original D is still liable for negligence of ambulance b/c he put P there in the first place.

Watson v Kentucky & Indiana Bridge and RR (KY 1910): D’s tank car leaked gas through D’s negligence. A disgruntled worker threw a match on the gas starting a fire. If the worker’s act had been negligent, D is still liable, but if it was malicious, then D could not have foreseen it, and is not liable.

R2T §449: Tortious or criminal acts the probability of which makes actor’s conduct negligent.If the likelihood that a 3rd person may act in a particular manner is the hazard or one of the hazards which makes the actor negligent, such an act whether innocent, negligent, intentionally tortious, or criminal does not prevent the actor from being liable for harm caused thereby.

Complication Two—complex scenariosCoincidence

Berry v Sugar Notch Borough (PA 1899): tree blew down on trolley car, injuring P. P was running the car in excess of the statutory speed, but this is not a cause—though D wants to argue that the excess speed meant that the car was under the tree when it blew down. Court refuses the argument as mere coincidence.

RescueWagner v International Ry (NY 1921): P’s cousin falls out of train, P goes searching for his body and is

injured. Is train liable? Held: danger invites rescue, P’s dangerous rescue does not necessarily eliminate D’s liability.

41

Page 42: Torts - University of Chicagoblsa.uchicago.edu/first year/TORTS/Torts-Epstein2002b.doc · Web viewTorts Introduction: Three categories of torts Intentional torts: where the defendant

Torts

D negligently creates a risk of harm A but harm B that is not foreseeable results to P.

Bomb and trigger theory—D controls bomb and trigger: Polemis.D sets stage with bomb, third party supplies trigger: Hines v. Garratt.D sets stage with bomb, P supplies trigger: Wagon Mound no. 1

Emotional distress:First line of defense: deny causal connectionSecond line of defense: even if D’s conduct is the cause in fact of P’s emotional harm, it is not the proximate cause.

Rationale for opposing recovery on emotional distress:1. damages too remote2. fear of false and ungrounded suits

Development:Step One: if there is no actual contact, there is no harm:

Mitchell v Rochester Ry (NY 1896): P found herself between D’s horsecars and was so afraid she fainted, then miscarried and suffered consequent illness. Expert testimony that mental shock was sufficient to produce these results. If D negligently manages the horse cars and P was not contributorily negligent, issue: can P recover? No recovery for fright b/c no immediate personal injury. If no recovery, then no liability. P’s miscarriage is not proximate b/c not ordinary and natural result and not expected result.

[Judge makes error by conflating proximate cause and judicial administrative duties. No, don’t want fraudulent cases, but fainting is probably sufficient evidence of harm. Today this would get to a jury.]

Step Two: zone of danger:Dulieu v White & Sons (KB 1901)—P gave birth prematurely after almost being run over by D’s horses while she was working behind the counter of her husband’s public house. Ct rejected requirement of physical injury, and damage can be proximate if it follows as direct and natural consequence of the injury.Problem: what is zone of danger? Behind, next to, in front of harm? How close?

Step Three: expanding zone of danger to those immediately affected without expanding it to those foreseeably affected:Dillon v Legg (CA 1968): D’s car hit and killed girl. Mother sues for nervous shock/mental pain, as done minor sister. Ct allowed sister’s claim b/c she was in zone of danger and feared for her own safety, but dismissed mother’s claim b/c her safety was not threatened. On appeal, Ct said that can’t deny mother’s claim b/c she was a few feet further away. Cts have rejected impact as standard and substituted zone of danger, but that necessitates artificial line-drawing. The only reason for zone of danger is that one within it fears impact, but impact has been rejected as a requirement.

--If P can’t recovery b/c of contributory negligence, then can’t recovery on a secondary suit for emotion distress either, b/c basis of claim has to be liability of D.--Don’t deny recovery b/c of fear of other fraudulent claims

--Dillon Test:1) whether P was located near the scene of the accident as contrasted with one who

was a distance away from it2) whether the shock resulted from a direct emotional impact upon P, as opposed to

hearing about it later3) whether P and V were closely related4) D should be able to reasonably foresee injury to P

42

Page 43: Torts - University of Chicagoblsa.uchicago.edu/first year/TORTS/Torts-Epstein2002b.doc · Web viewTorts Introduction: Three categories of torts Intentional torts: where the defendant

Torts

5) Does D owe P a duty of careUse objective standard of foreseeability on a case-by-case basis

Dillon creates an arbitrary administrative line in attempt to address same two issues: adminstrability & substantive legal issue.Later cases struggled with the Dillon criteria:

What if mother arrived on the scene only second later: Tobin v Grossman (NY 1969)—denied b/c didn’t witness. Also worried about where to draw the line with relationship to V.

What is a close relationship: Elden v Sheldon (CA 1988): denied recovery to unmarried cohabitant of V who witnessed accident.

What if mother didn’t witness accident: Thing v La Chusa (CA 1989): denied, ct expressed a Bright Line Rule for recoverability for emotional distress:

1) P is closely related to V2) P is present at the scene and is aware of injury to V3) As a result P suffers emotional distress beyond that which would be anticipated

in a disinterested witnessLater cases in CA eschew foreseeability and ask instead whether D has assumed some direct duty to P. These have been strictly limited. E.g., no direct duty to parents of child harmed by negligently filling prescription; but mother is a direct victim when malpractice caused her child brain damage during birth.

Outside California:Some states retain zone of danger, but most have gone beyond it.Have to draw a distinction between actionable emotional distress and the usual distress caused by, e.g., hearing about a death, which is not actionable.

At risk Ps: when Ps have a higher chance of getting cancer b/c of drugs and toxic torts, but haven’t gotten it yet, can’t get damages for the distress, but can get recovery for medical monitoring costs.

Epstein interpretation: avoid making up special rules to deal with special cases. Find a way to make physical injury and emotional distress parallel.:

1. it’s a jury question as long as there’s not a freakish concatenation of events.2. as level of harm per case is reduced, and number of harms increases, abandon tort

system.3. ask if the harm is within the mere fright rule—that’s not enough for recovery. (Like

reciprocal harms in nuisance: live and let live.)4. sort out the background harms (no recovery) from the foreground harms (recovery if Ps

are closely enough related to harm.) [Similar to public nuisance and who can sue privately.]

F. Direct victimsMolien v Kaiser Foundation Hospital (CA 1980): Dr. negligently told P’s wife she had syphilis. Her suspicion of her husband led to a divorce. P’s suit for emotional distress allowed on grounds that P was a direct and foreseeable victim.

43

Page 44: Torts - University of Chicagoblsa.uchicago.edu/first year/TORTS/Torts-Epstein2002b.doc · Web viewTorts Introduction: Three categories of torts Intentional torts: where the defendant

Torts

Affirmative DutiesDistinction between misfeasance (active misconduct working positive injury) & nonfeasance

(passive inaction)a. No settled distinction between failure to prevent harm and causing harm by

nonaction.1. Example: policeman sees flaming car in ditch, calls fire dept and directs traffic but does not

rescue people in car, one person dies. Is this negligence or nonfeasance?2. Courts often hold that negligence includes: the omission to do something a reasonable person

would do as well as the doing of something a prudent person would not do.b. Opposition misfeasance-nonfeasance (as seen in strict liability regime)

1. Misfeasance: D acts when he applies force to the person or property of a stranger or creates a dangerous condition that causes harm

2. Nonfeasance: D has not used force or created some dangerous situationi. Failure to warn

G. Question = finding an independent source for the duty to actA. to analyze duty to act specify:

1. the duty if any imposed on D2. contingencies against which D must guard

a. his own actsb. acts of Godc. acts of third parties

3. the associated standard of liabilitya. strict liabilityb. negligencec. indifference or willful neglect

H. RULE: there is no general duty to take affirmative action. a. Buch v Amory Manufacturing : actionable negligence is the neglect of a legal duty. D is

not liable unless he owed to P a legal duty which he neglected to performb. A moral duty is not legally enforceablec. If D sees that P is in danger, and fails to render assistance (even though D could do so

easily and safely), D is not liable for refusing to assist.d. The fact that D foresees harm to a particular individual from his failure to act does not

change the rule.1. Example: D sees blind person about to step in front of car. D could prevent injury without harm

to himself, without delaying his progress, but D does nothing. D is not liable.e. Rationale:

1. D is responsible for the consequences of his own actions, but not a conscripted saviour against the misfortunes that befall others

i. Maximize individual autonomy and choice by minimizing coercive legal interactions imposed on unsuspecting bystanders

2. Administrability: it would be unworkable to impose liability even where people can rescue at low cost to themselves and high benefit to others.

3. There might be a negative reaction reducing the willingness to rescue voluntarily b/c of fear of liability

4. Asymmetry: Reckless people would have an incentive to be more reckless & careful people have more duties placed on them

44

Page 45: Torts - University of Chicagoblsa.uchicago.edu/first year/TORTS/Torts-Epstein2002b.doc · Web viewTorts Introduction: Three categories of torts Intentional torts: where the defendant

Torts

5. Difficulty of prosecuting the lawsuit: who had the duty, what level of duty, would the rescue have made a difference.

i. Which of the 50 people on the bridge has the duty to throw down the rope?f. Other approaches:

1. Vermont statute —requires assistance when D knows of grave physical danger to another and can assist without danger to himself.

2. Shapo: law should impose affirmative duty to act in an emergency when reasonable people would do so, or when D has special power to prevent harm with little effort.

3. Bender: Law should reflect virtue of compassion.4. Ames: D should act when the rescue could be done with little or no

inconvenience to Di. Epstein: if rescue is cheap, someone will undertake it anyway.

5. Provide restitution to rescuer for benefits conferred on P—but how to measure benefits?

Exceptions to general rule of nonfeasance—turn on issue of duty of D to P:Four categories:

1. D or his instrumentalities, innocently or not, created risks or caused harm to P.2. D is in a special relationship to P that is deemed to create a duty of care that

encompasses affirmative action3. D takes affirmative action that is either cut short or performed negligently

4. D has assumed a duty of affirmative care by action or promise that evinces such an assumption

NB: Even in these cases, courts do not impose strict liability, but only duty to act when a reasonable person would.

A. Torts exceptionsRule: D who has created a risk and is in a position to mitigate its effects has a duty of care

toward Pa. P’s action lies only for the new harm that D’s remedial action could have

forestalled or preventedb. D has no duty when he does not know of P’s injury or peril, or, having

discovered it, has no opportunity to institute cost- or life-saving measures1. Creates line-drawing problems

c. D has to act even when P is guilty of contributory negligence d. D is liable when D does not harm P but makes it more difficult for P to get aide

from someone elseRationale: Give D incentive to take action to mitigate or forestall P’s injury

II. RescueA. There is no affirmative duty to rescue, assist, aid or protect a stranger

from any danger.B. But, duty created by D when D:

1. innocently causes harm

45

Page 46: Torts - University of Chicagoblsa.uchicago.edu/first year/TORTS/Torts-Epstein2002b.doc · Web viewTorts Introduction: Three categories of torts Intentional torts: where the defendant

Torts

2. both victim and D are negligent, victim is injured, D is liable to help

3. D undertakes to improve condition of victim by starting rescuea. Don’t penalize D for trying to help

i. Muffed rescue won’t lead to liability unless D prevents officials from doing their job

ii. Society can’t ask rescuer to do more than they are able

b. D starts then abandons rescuei. D is liable if he leaves V in worse circumstances

than before he starts rescueii. D is probably not liable if he leaves V worse off

than V would have been if he had been rescued, but not worse off than he was before D started rescue and abandoned it

I. R2T§322 Duty to aid another harmed by actor’s conduct: If the actor knows or has reason to know that by his conduct, whether tortious or innocent, he has caused such bodily harm to another as to make him helpless and in danger of further harm, the actor is under a duty to exercise reasonable care to prevent such further harm.

i. D has opportunity to minimize the risk before harm actually happensD. R2T §323: Negligent performance of undertaking to render services: One

who undertakes, gratuitously or for consideration, to render service to another which he should recognize as necessary for the protection of the other’s person or things, is subject to liability to the other for physical harm resulting from his failure to exercise reasonable care to perform his undertaking, if:b) his failure to exercise such care increases the risk of such harm, orc) the harm is suffered b/c of the other’s reliance upon the undertaking.

E. R2T §324: Duty of one who takes charge of another who is helpless: One who, being under no duty to do so, takes charge of another who is helpless adequately to aid or protect himself is subject to liability to the other for any bodily harm caused to him by:

a) the failure of the actor to exercise reasonable care to secure the safety of the other while within the actor’s charge, or

b) the actor’s discontinuing his aid or protection, if by so doing he leaves the other in a worse position than when the actor took charge of him.

F. R2T §327: any person who knows or has reason to know that a 3rd person is giving or is ready to give another aid necessary to prevent physical harm to and endangered person is tortiously liable if he negligently prevents or disables the 3rd person from giving such aid.

CASES: (everything turns on who has a right to be where)Buch v Amory Manufacturing Co (NH 1897): 8 year old boy trespasses onto D’s mill. D’s overseer tried to

shoe him out, but P didn’t speak English. P got hand crushed in machine as his brother tried to show him how to work it. Issue: what duty did D owe to the trespasser? Held: Owner owes no duties to a trespasser except to avoid personal violence.

Mangan v Atterton (Ex 1866): whitesmith left his machine unattended and without locking up rollers, boys playing with it, P got hand crushed. On appeal found for D b/c he owed no duty to the child.

46

Page 47: Torts - University of Chicagoblsa.uchicago.edu/first year/TORTS/Torts-Epstein2002b.doc · Web viewTorts Introduction: Three categories of torts Intentional torts: where the defendant

Torts

Yania v Bigan (PA 1959): P and D were strip miners. P visits D’s mines to talk business. D cajoled P into jumping into cut filled with water. P drowned D did nothing. D not liable b/c P knew the danger and as an adult was capable of withstanding verbal taunting.

Montgomery v National Convoy & Trucking Co (SC 1937): D’s trucks stalled on icy road without any negligence, blocking the road. P’s car came over crest of hill and was unable to stop b/c of ice. D failed to place warning at top of hill even though they knew no car would be able to stop once it was in position to see the roadblock. Held: Ds clearly owed duty to other highway users in such a way as to prevent harm. Putting flares around the trucks was not sufficient to warn.

[If you took this as a strict liability issue, D knows what his duty is and will take steps to mitigate his own liability—so you have no problem deciding case.]

Black v New York, NH & H R R (MA 1909): conductor carries drunk off train at his stop, sets him on stairs. P falls down stairs, injuring himself. Held: D was negligent in leaving P where he would be likely to be harmed.

Zelenko v Gimbel Bros (NY Sup Ct 1935): D moves P from public to private place, doesn’t provide medical care, which P might have gotten if she had remained in the public place. Rule: if a D undertakes a task, even if under no duty to undertake it, D must not omit to do what an ordinary man would do in performing the task.

Soldano v O’Daniels (CA App 1983): V was in imminent peril of being shot, when another person ran across the street to D’s bar and asked to use the phone to call the police. D refused, V was killed. Held: D, though not required to aid himself, was required to allow the police to be called.

B. Contracts exceptionsI. Gratuitous undertakings—issue is whether or not D had assumed a duty of care.

1. Executory contracts:ЉЉЉЉЉЉЉЉ赘ꍪЉЉЉЉЉЉЉЉЉЉЉЉЉЉЉЉЉЉЉЉЉЉЉЉЉЉЉЉЉЉЉЉЉЉЉЉЉЉЉЉЉЉЉЉΪ௲ЉЉЉЉЉЉЉЉЉЉЉЉЉЉЉЉЉЉЉЉЉЉЉЉЉЉЉЉЉЉЉЉЉЉЉЉЉЉЉЉЉЉЉЉЉЉЉЉЉЉЉЉЉЉЉЉЉЉЉЉЉЉЉЉЉЉЉЉЉࢦ ЉЉ⫤ 葂ЉЉЉЉЉ碈膘ЉЉЉЉЉЉЉЉЉЉЉЉЉЉЉЉЉЉЉЉЉЉЉЉЉЉЉЉЉЉЉЉЉЉЉЉЉЉЉЉЉЉЉЉЉЉЉЉЉЉЉЉЉЉЉЉЉЉЉЉЉЉЉЉЉЉЉЉЉЉЉЉЉЉЉЉЉЉЉЉЉЉЉЉЉЉЉЉЉЉЉЉЉЉЉЉЉЉЉЉЉЉЉЉЉЉЉЉЉЉЉЉЉЉЉЉЉЉЉЉЉЉЉЉЉЉЉЉЉЉЉЉЉЉЉЉЉЉЉЉЉЉЉЉЉЉЉЉЉЉЉЉЉЉЉЉЉЉЉЉЉЉЉЉЉЉЉЉЉЉЉЉЉЉЉЉЉЉЉЉЉ閦ꁢЉЉЉЉЉЉЉЉЉЉЉЉЉЉЉЉ柮㝾ЉЉЉЉЉЉЉЉЉЉЉЉЉЉЉЉЉ峦눈ЉЉЉЉЉЉ Old theory —no liability until D begins to act for P’s benefiti. Thorne v Deas (NY 1809): D & P are co-owners of a ship.

D promises gratuitously to insure the ship, P sets sail. D fails to insure, ship is wrecked. D is not liable b/c promise was unsupported by consideration and executory.

ii. Coggs v Bernard (KB 1703): D moved casks of brandy owned by P and in process some split open a brandy was lost. D moved for arrest of judgment b/c there was no consideration and he was not doing this as his job. Judgment for P b/c D had begun to act. For liability, an agreement to perform can’t be executory but rather actual entry upon the thing, taking the trust upon himself. The owner’s trusting him with the goods is sufficient consideration.

ЉЉЉЉЉЉЉЉ赘ꍪЉЉЉЉЉЉЉЉЉЉЉЉЉЉЉЉЉЉЉЉЉЉЉЉЉЉЉЉЉЉЉЉЉЉЉЉЉЉЉЉЉЉЉЉΪ௲ЉЉЉЉЉЉЉЉЉЉЉЉЉЉЉЉЉЉЉЉЉЉЉЉЉЉЉЉЉЉЉЉЉЉЉЉЉЉЉЉЉЉЉЉЉЉЉЉЉЉЉЉЉЉЉЉЉЉЉЉЉЉЉЉЉЉЉЉЉࢦ ЉЉ⫤ 葂ЉЉЉЉЉ碈膘ЉЉЉЉЉЉЉЉЉЉЉЉЉЉЉЉЉЉЉЉЉЉЉЉЉЉЉЉЉЉЉЉЉЉЉЉЉЉЉЉЉЉЉЉЉЉЉЉЉЉЉЉЉЉЉЉЉЉЉЉЉЉЉЉЉЉЉЉЉЉЉЉЉЉЉЉЉЉЉЉЉЉЉЉЉЉЉЉЉЉЉЉЉЉЉЉЉЉЉЉЉЉЉЉЉЉЉЉЉЉЉЉЉЉЉЉЉЉЉЉЉЉЉЉЉЉЉЉЉЉЉЉЉЉЉЉЉЉЉЉЉЉЉЉЉЉЉЉЉЉЉЉЉЉЉЉЉЉЉЉЉЉЉЉЉЉЉЉЉЉЉЉЉЉЉЉЉЉЉЉЉ閦ꁢЉЉЉЉЉЉЉЉЉЉЉЉЉЉЉЉ柮㝾ЉЉЉЉЉЉЉЉЉЉЉЉЉЉЉЉЉ峦눈ЉЉЉЉЉЉ Promissory Estoppel theory —where P has relied to her detriment on D’s promise, D can’t get out of liability for performance whether or not he has begun to perform. Promise can be implied-in-fact.i. Erie RR v Stewart (6th Cir 1930): P = passenger in a truck

struck by train at crossing where guard was not doing his duty in giving warning. Held: Where custom established usage, RR had created a positive duty for itself by establishing a standard of due care upon which the public relied. Once established the duty can’t be discontinued without warning.

47

Page 48: Torts - University of Chicagoblsa.uchicago.edu/first year/TORTS/Torts-Epstein2002b.doc · Web viewTorts Introduction: Three categories of torts Intentional torts: where the defendant

Torts

ii. Marsalis v La Salle (La App 1957): P scratched by D’s cat in D’s store while P was shop[ing there. P asked D to keep cat locked up to be sure it didn’t have rabies. D agreed, but after 5 days, cat escaped. So P was vaccinated for rabies, P allergic to treatment. Rule: one who voluntarily undertakes to care for, or to afford relief or assistance to an ill, injured, or helpless person is under a legal duty to use reasonable care and prudence in what he does. Held: P relied upon D to fulfill a voluntary duty, D obligated to use reasonable care; D did not take any precautions to keep cat locked up and is liable for P’s damages.

ЉЉЉЉЉЉЉЉ赘ꍪЉЉЉЉЉЉЉЉЉЉЉЉЉЉЉЉЉЉЉЉЉЉЉЉЉЉЉЉЉЉЉЉЉЉЉЉЉЉЉЉЉЉЉЉΪ௲ЉЉЉЉЉЉЉЉЉЉЉЉЉЉЉЉЉЉЉЉЉЉЉЉЉЉЉЉЉЉЉЉЉЉЉЉЉЉЉЉЉЉЉЉЉЉЉЉЉЉЉЉЉЉЉЉЉЉЉЉЉЉЉЉЉЉЉЉЉࢦ ЉЉ⫤ 葂ЉЉЉЉЉ碈膘ЉЉЉЉЉЉЉЉЉЉЉЉЉЉЉЉЉЉЉЉЉЉЉЉЉЉЉЉЉЉЉЉЉЉЉЉЉЉЉЉЉЉЉЉЉЉЉЉЉЉЉЉЉЉЉЉЉЉЉЉЉЉЉЉЉЉЉЉЉЉЉЉЉЉЉЉЉЉЉЉЉЉЉЉЉЉЉЉЉЉЉЉЉЉЉЉЉЉЉЉЉЉЉЉЉЉЉЉЉЉЉЉЉЉЉЉЉЉЉЉЉЉЉЉЉЉЉЉЉЉЉЉЉЉЉЉЉЉЉЉЉЉЉЉЉЉЉЉЉЉЉЉЉЉЉЉЉЉЉЉЉЉЉЉЉЉЉЉЉЉЉЉЉЉЉЉЉЉЉЉЉ閦ꁢЉЉЉЉЉЉЉЉЉЉЉЉЉЉЉЉ柮㝾ЉЉЉЉЉЉЉЉЉЉЉЉЉЉЉЉЉ峦눈ЉЉЉЉЉЉ R2T §323: Negligent performance of undertaking to render services: One who undertakes, gratuitously or for consideration, to render service to another which he should recognize as necessary for the protection of the other’s person or things, is subject to liability to the other for physical harm resulting from his failure to exercise reasonable care to perform his undertaking, if:

a) his failure to exercise such care increases the risk of such harm, orb) the harm is suffered b/c of the other’s reliance upon the undertaking.

ЉЉЉЉЉЉЉЉ赘ꍪЉЉЉЉЉЉЉЉЉЉЉЉЉЉЉЉЉЉЉЉЉЉЉЉЉЉЉЉЉЉЉЉЉЉЉЉЉЉЉЉЉЉЉЉΪ௲ЉЉЉЉЉЉЉЉЉЉЉЉЉЉЉЉЉЉЉЉЉЉЉЉЉЉЉЉЉЉЉЉЉЉЉЉЉЉЉЉЉЉЉЉЉЉЉЉЉЉЉЉЉЉЉЉЉЉЉЉЉЉЉЉЉЉЉЉЉࢦ ЉЉ⫤ 葂ЉЉЉЉЉ碈膘ЉЉЉЉЉЉЉЉЉЉЉЉЉЉЉЉЉЉЉЉЉЉЉЉЉЉЉЉЉЉЉЉЉЉЉЉЉЉЉЉЉЉЉЉЉЉЉЉЉЉЉЉЉЉЉЉЉЉЉЉЉЉЉЉЉЉЉЉЉЉЉЉЉЉЉЉЉЉЉЉЉЉЉЉЉЉЉЉЉЉЉЉЉЉЉЉЉЉЉЉЉЉЉЉЉЉЉЉЉЉЉЉЉЉЉЉЉЉЉЉЉЉЉЉЉЉЉЉЉЉЉЉЉЉЉЉЉЉЉЉЉЉЉЉЉЉЉЉЉЉЉЉЉЉЉЉЉЉЉЉЉЉЉЉЉЉЉЉЉЉЉЉЉЉЉЉЉЉЉЉЉ閦ꁢЉЉЉЉЉЉЉЉЉЉЉЉЉЉЉЉ柮㝾ЉЉЉЉЉЉЉЉЉЉЉЉЉЉЉЉЉ峦눈ЉЉЉЉЉЉ Modern K’s often limit liability based on privity.i. Moch Co. v. Rensselaer Water Co. (NY 1928): D

contracted with city of Rensselaer to furnish city with water. While K was in effect, a building caught fire, spread to and destroyed P’s warehouse. D failed to provide sufficient water to the hydrants to extinguish the fire. Point: city paid for the water, P only contributed in the form of taxes. City had the K with the water company. City had no legal duty to protect citizens from fire. Extending D’s liability to those not in privity of K means D’s liability is being increased without its compensation being increased.

1. [Contract out of consequential damages and use system design to best deal with utilities and insurance situations.]

C. Special Relationships1. Different situation than stranger-rescue cases b/c the special

relationship identifies who is responsible for the well-being of another.i. Can’t use excuse that law is thrusting positive obligation of care

on an unwitting bystander.2. Covers all cases in which D is put in charge of P—D is

assuming some charge or protective role over P directly or over the premises P uses.i. Carrier-passenger: utmost care, but doesn’t have to be

armed guardsii. Innkeeper-guest

iii. Landowner-invitee [see below]: shopping malls, hotels

48

Page 49: Torts - University of Chicagoblsa.uchicago.edu/first year/TORTS/Torts-Epstein2002b.doc · Web viewTorts Introduction: Three categories of torts Intentional torts: where the defendant

Torts

a. Business and invitee of business: reasonable care to discover dangers and to warn or protect

b. Example: a landowner whose invitee becomes ill may have a duty to summon aid or at least provide shelter until invitee improves.

iv. Custodian-ward—schools, hospitals, universities1. Example: operator of day-care center has duty to use

reasonable care to obtain medical care for ill or injured child in his care.v. Employer-employee

vi. Parent-minor childvii. Doctor patient

viii. Courts may find other relationships: social companions, landlord-tenant

3. Level of care = reasonable care under the circumstances (negligence standard)

4. Contingencies against which D must guard:i. D must take reasonable steps to free premises under

his control from latent hazardsii. D must inspect and repair and correct dangers quickly

when brought to his attentioniii. D must maintain facilities to help rescue P in case

danger arisesiv. Newer standard: duty arises when there is a probable

and predictable danger 1. Older rule: the first event of its kind is not foreseeable,

so there is no liability. Often replaced with a totality of the circumstances approach, which is giving way to requirement that three be a high degree of foreseeability of the crime to indicate that D should be taking precautions.

2. Kline v 1500 Massachussetts Ave Apartment Corp (DC Cir 1970): tenant robbed and assaulted in hallway of her apt building. Landlord has duty of reasonable care in providing safe building b/c he is he best risk avoider and only landlord has power to take action in common areas. Doctrine: tenant’s ability to provide for own safety is limited b/c tenant has submitted to partial control of landlord, the duty is imposed on the one with control and thus power to act to take reasonable precautions to protect the tenant. Provided that the violence is 1) foreseeable, and 2) not by an employee of the landlord.

3. Weirum v. RKO General Inc . (Cal 1975): Disk jockey promotional stunt caused listeners to drive recklessly to win prize. Special imposition of duty under theory that you are liable for actions of 3rd parties if you inspire them to act in dangerous ways.

4. Duty of care is what is reasonable under the circumstances

5. Problem : causation can be very difficult to prove. To have action against landlord for injuries by intruders, tenant has to show 1) causal link between landlord’s culpable failure to provide security and the injuries, but this is usually only possible if intruders can be identified, so P has burden of proof of establishing proximate cause but can do so with inference.

5. R2T §357: duty of lessor of land for harms to lessee or guests: A lessor of land is subject to liability for physical harm caused to his lessee and others upon the land with the consent of the lessee or his sublessee by a condition of disrepair existing before or arising after the lessee has taken possession if:

a) the lessor, as such, has contracted by a convenant in the lease or otherwise to keep the land in repair, and

49

Page 50: Torts - University of Chicagoblsa.uchicago.edu/first year/TORTS/Torts-Epstein2002b.doc · Web viewTorts Introduction: Three categories of torts Intentional torts: where the defendant

Torts

b) the disrepair creates an unreasonable risk to persons upon the land which the performance of the lessor’s agreement would have prevented, and

c) the lessor fails to exercise reasonable care to perform his K.

D. Prevention of harm by third parties1. D is under a duty to control an dangerous

person with whom he has special relationship—often an institutional custodial relationship.

2. D has duty to 3rd persons with whom he has no relationship to prevent P from causing them harm.

a. Duty to warn/duty to confine, duty to alert public authorities--whatever is reasonablei. Problem: how to decide when it

is proper to warn authorities in an institutional setting? Establish internal review mechanism.

b. Liability limited to negligencei. No liability if victim is not

identifiable1. Thompson v County of

Alameda (CA 1980): juvenile released from custody even though he said he would kill child if released. Within 24 hours of release, he kills P’s son. No liability for warning b/c no targeted victim.

c. standard of care: reasonable under the circumstancesi. Problem: if you can’t prove

deviation from customary standard, causation is hard to show. 1. Figure out what could have

been done ex ante to prevent harm.2. Think about it from P’s

perspective: what did they know, did they assume risk.3. R2T §315 special relationships: There is no

duty so to control the conduct of a 3rd person as to prevent him from causing physical harm to another unless:

a) a special relationship exists between the actor and the 3rd person which imposes a duty upon the actor to control the 3rd person’s conduct, or

b) a special relationship exists between the actor and the other which gives the other a right to protection.

Tarasoff v Regents of Univ of California (CA 1976): Poddar killed Tarasoff, 2 months before he had told his psychologist he planned to kill her. Psychologist had police detain him but he seemed sane. No further action taken to detain Poddar. No one warned Tarasoff. Rule: D only has to control actions of a 3rd person if there is a special relationship either with 3rd person or with the person to be protected. Held: psychiatrist could be held liable under negligence to warn intended victim or confine assailant until danger had passed.

50

Page 51: Torts - University of Chicagoblsa.uchicago.edu/first year/TORTS/Torts-Epstein2002b.doc · Web viewTorts Introduction: Three categories of torts Intentional torts: where the defendant

Torts

E. Owner’s and Occupier’s LiabilityI. Classificatio

n of entrants onto landi. Trespass

ers—a person who enters or remains upon land in the possession of another without a privilege to do so created by the possessor’s consent or otherwise.

ii. Invitees—persons who is invited onto the premises by the owner as a business visitor for some business purpose of mutual benefit to both parties, or a public invitee who is invited to some public place not only for business purposes but also for a purpose for which the land is held open to the public.

iii. Licensees—persons who enter the premises with the permission of the owner for their own benefit and who can be ordered to leave at any time. (Social guest, not normally associated with business purposes of owner of the property.)

II. Innocent Trespassers

i. Duty of care: No duty to take care or to warn of dangers.

a. Occupier is liable only where the injury is due to some willful act involving something more than the absence of reasonable care, e.g. intentional act to harm the trespasser, or reckless disregard.

b. Standard case:Robert Addie & Sons v. Dumbreck (A.C. 1929): Coal ash haulage system, dangerous and not well protected from public including children, but D warned public to stay away. Small child killed in wheel mechanism. Child held to be trespasser. Proprietor has no duty to fence his land under sanction that if he doesn’t the public become licensees.

c. Wanton & willful exception—once D knows there is a trespasser in danger, he has to take affirmative act at least not to increase harm:Excelsior Wire Rope v. Callan (A.C. 1930): two children had hands crushed in haulage mechanism. Held to be trespassers, but ct found D liable. Distinguished from Addie b/c D’s employees acted in reckless disregard of Ps’ welfare.Gould v DeBeve (D.C. Cir 1964): child fell out of apartment window b/c of defective screen, but child and parent weren’t supposed to be living there, so technically a trespasser. Held P was trespasser, but D was guilty of wanton and willful misconduct in ignoring statutory obligation to replace the defective screen.

d. Attractive nuisance—duty to child trespassers Allows infant trespassers to recover when lured onto D’s property by

some tempting condition created and maintained by D. (E.g. railway turntables, explosives, smoldering fires, rickety structures.)

o Two criteria: 1) foreseeable risk of trespass because 2) dangerous condition was sufficiently enticing to cause children to trespass in the first place

o Problem is determining limits. Doesn’t extend to: rivers, creeks, ponds, wagons, axes, plows, woodpiles, haystacks, etc.

R2T § 339: Artificial Conditions highly dangerous to trespassing children.A possessor of land is subject to liability for physical harm to children trespassing thereon caused by an artificial condition upon the land if

51

Page 52: Torts - University of Chicagoblsa.uchicago.edu/first year/TORTS/Torts-Epstein2002b.doc · Web viewTorts Introduction: Three categories of torts Intentional torts: where the defendant

Torts

a) the place where the condition exists is one upon which the possessor knows or has reason to know that children are likely to trespass, and

b) the condition is one of which the possessor knows or has reason to know and where he realizes or should realize will involve an unreasonable risk of death or serious bodily harm to such children, and

c) the children b/c of their youth do not discover the condition or realize the risk involved in intermeddling with it or in coming within the area made dangerous by it, and

d) the utility to the possessor of maintaining the condition and the burden of eliminating the danger are slight as compared with the risk to children involved, and

e) the possessor fails to exercise reasonable care to eliminate the danger or otherwise to protect the children.

generally not held to apply to natural conditions. possessor under no duty to investigate the land to determine whether

trespassing children are present. Just knows or has reason to know. Clause c=assumption of risk exception. If child is old enough to know

the danger, no liability.III. Invitees—

take affirmative actions to warn and maintain and repair premises.i. Contract

-based relationshipii. Too

many people to warn them each, so owner has to act to make premises safea. When one patron creates a danger for another, owner liability is premised on

how much time the commercial entity had to identify and correct danger—owner has to take reasonable care, but system builds in some slack.

IV. Licensees—duty to warn

i. Social relationship—guests are licensees for some parts of house and trespassers in others

ii. If owner agrees to allow atypical guests onto premises, owner assumes duty of protecting them.

V. Differentiating between licensee and invitee:

i. Use premise-based test founded in understood social uses.

ii. Focus on nature of the premises rather than nature of the visit: those who run business premises, or premises to which the public is generally invited, are subject to the rules for invites; those who maintain private or residential premises are subject to rules for licensees.

iii. As the nature of the premise changes, the nature of the duty changes.

VI. Licensees and invitees create informal Ks with owner. Tort law is filling in the gaps after an injury has occurred.

52

Page 53: Torts - University of Chicagoblsa.uchicago.edu/first year/TORTS/Torts-Epstein2002b.doc · Web viewTorts Introduction: Three categories of torts Intentional torts: where the defendant

Torts

VII. Common law rule for non-liability in natural conditions: rock slides, falling trees that occur without fault of owner of property.

i. Where acreage is extensive, don’t hold possessor liable for duty to warn beyond house and yard b/c owner has no comparative advantage over guest

VIII. Abandoning tripartite distinction:Rowland v Christian (CA 1968): D invited P to her house, porcelain handle of faucet broke severing nerves and tendons in P’s hand. D knew of the crack, but didn’t warn D of danger. Danger was not obvious. Ct decides to abandon 3 classifications of visitors to land b/c they are not reasonable, and to apply general liability principles of duty owed not to harm others.Statutes have taken up what Rowland changed by providing extensive protections to

landowners.

53

Page 54: Torts - University of Chicagoblsa.uchicago.edu/first year/TORTS/Torts-Epstein2002b.doc · Web viewTorts Introduction: Three categories of torts Intentional torts: where the defendant

Torts

Traditional Strict LiabilityI. Conversion

Definition of Conversion: a distinct act of dominion wrongfully exerted over another’s personal property in denial of or inconsistent with his title or rights therein, without the owner’s consent and without lawful justification.

P only has to allege1. P’s ownership or right to possession of property at time of the conversion2. D’s conversion by a wrongful act or disposition of P’s property rights3. damages

Strict liability tort: 1. D need have neither knowledge nor intent. 2. Based solely on the interference. 3. Negligence, malice are not issues.

Remedies: restore property or forced sale.Moore v The Regents of the University of California (Cal App 1988/CA 1990): Drs take P’s spleen b/c he

has cancer. Discover his cells are unique, develop the cell line, patent it and sell it for billions. Continue for 7 years to monitor P for research reasons and to take tissue samples. Was there conversion? Appellate court says yes b/c P owned tissues when they were taken from him. Supreme Court says no b/c body parts are unowned. To claim conversion, P has to have had ownership or right of possession before alleged conversion. Try to deal with issue by recommending informed consent agreements.

Trespass to chattel: intentional interference with a person’s use or possession of a chattel.D only has to pay damages, not full value of the property.Trespass lies when D has taken possession of P’s goods without claiming ownership of them.

II. AnimalsAnimals causing:

1. damage to persons strict liability, at least for animals classified as dangerous by nature

a. for animals tame by nature, owner usually only liable for negligence but for strict liability if an animal has shown dangerous propensities.

a) Does this distinction make sense? Since most harm caused by dangerous animals, why not make a straight strict liability standard for all animals.

Baker v Snell (KB 1908): woman bitten by dog owned by her master and known to the master to be vicious and given to bite. Issue: “If a man keeps an animal whose nature is ferocious, or an animal of a class not generally ferocious, but which is known to the owner to be dangerous, is the owner of that animal liable only if he neglects his duty of keeping it safe or is negligent in the discharge of that duty, or is he bound to keep it secure at his peril?” Ct holds the last. If you keep the animal knowing it is dangerous, you are strictly liable b/c you took the risk of harm.

2. damage to property owner liable for damage to real property and animals grazing there. Problem = setting limits of liability.

R2T §504: strict liability of possessor of trespassing livestock does not extend to harm1. not reasonably to be expected from the intrusion2. done by animals straying onto abutting land while driven on the highway, or3. brought about by the unexpected operation of a force of nature, action of another

animal or intentional, reckless or negligent conduct of a 3rd personAffirmative defenses in animal cases

4. P incited animal to attack him5. P assumed risk (often = workmen’s comp cases or visitors

to parks)6. P was stupid

54

Page 55: Torts - University of Chicagoblsa.uchicago.edu/first year/TORTS/Torts-Epstein2002b.doc · Web viewTorts Introduction: Three categories of torts Intentional torts: where the defendant

Torts

Bailor is not liable if animal causes harm while in possession of bailee Zoos not liable in strict liability but under negligence.

III. Hazardous activities—see Rylands v. FletcherA. Liability flows from the choice to conduct the dangerous activity in the

first place.i. If injury results, precautions are not defense

ii. Producer of danger is best cost-avoider b/c can get insurance and pass costs onto customers or find other ways to do the job that aren’t so dangerous.

iii. Want an anticipatory regime—know in advance what it dangerous.

Epstein test for hazardous activities: if you have to have a permit to do it, it’s hazardous.B. At issue is not at the property of the hazardous substance but rather what

D does with it—at his activities.i. E.g.: blasting, large artificial ponds for retaining mining wastes, crop-dusting,

fumigation, storage of large quantities of gasoline, rockets, experimental aircraft, use of radioactive materials.

C. No thin skull exception: no strict liability if there would have been no harm except for P’s abnormal sensitivity.

D. RULESI.General principle R2T § 519

1. one who carries on an abnormally dangerous activity is subject to liability for harm to the person, land, or chattels or another resulting from the activity, although he has exercised the utmost care to prevent the harm

2. this strict liability is limited to the kind of harm, the possibility of which makes the activity abnormally dangerous

a. D isn’t strictly liable for harms unrelated to the activity. E.g. if a truck carrying hazardous chemicals runs someone over, the driver is liable in negligence and the nature of the freight is irrelevant.

i. Causation problems: how far to ramifications of the dangerous activity extend?Madsen v East Jordan Irrigation Co (UT 1942): D’s blasting caused the mother mink on P’s farm to eat their young. “He who fires explosives is not liable for every occurrence following the explosion which has a semblance of connection to it.”

Six-factor Test: One factor alone is usually not sufficient, but don’t have to have all of them. Epstein doesn’t like this test.

J. Abnormally dangerous activities R2T §520Consider following factors to decide if activity is abnormally dangerous

1. existence of a high degree of risk of some harm to person, land or chattels of another

2. likelihood that the harm that results from it will be great3. inability to eliminate the risk by the exercise of reasonable care

a. when safety cannot be attained by the exercise of due care there is reason to regard the danger as an abnormal one

4. extent to which the activity is not a matter of common usagea. e.g. driving is hazardous but so common that it doesn’t meet this criterion

5. inappropriateness of the activity to the place where it is carried on6. extent to which its value to the community is outweighed by its dangerous

attributes

55

Page 56: Torts - University of Chicagoblsa.uchicago.edu/first year/TORTS/Torts-Epstein2002b.doc · Web viewTorts Introduction: Three categories of torts Intentional torts: where the defendant

Torts

a. if the utility of carrying on the activity doesn’t justify the risk it creates, it may be negligent even to do it

b. D does not have to get a monetary benefit from the activityc. If the value to the community is great, the risk will not be considered abnormal

i. Cts disagree about importance of this factor b/c utility is a subjective measure R2T §522: Contributing actions

One carrying on an ultrahazardous activity is liable for harm even if harm is unexpectedly caused by the innocent, reckless or negligent conduct of a 3rd person, the action of an animal, or a force of nature.

R2T §523: Assumption of risk by P from harm from an abnormally dangerous activity bars his recovery for the harm.

R2T §520A: Ground damage by aircraft Aircraft is liable to harm to objects/persons on the ground caused by aircraft or falling parts even though D exercised utmost care to prevent it.

Doesn’t really use six factor test.CASES:

Spano v Perini Corp (NY 1969): P’s garage and car damaged by D’s blasting.Rule: one who engages in blasting must assume responsibility and be liable without fault for

any injury he causes to neighboring property. We can infer per se negligence if harm caused by blasting

Rationale: 1. D is best risk bearer, risk avoider2. Absolute liability will encourage blasters to find alternative methods

Indiana Harbor Belt RR v American Cyanamid Co (7th Cir 1990): D put hazardous chemical into train car, which was carried to P’s switching yard, where it was found to be leaking. P sues D in strict liability to recover cost of clean-up. Ct refuses to apply s.l. b/c 6 R2T §520 criteria not met. It wouldn’t necessarily be safer to route train away from metropolitan areas; it is possible to be safe using reasonable care. [Posner gets it wrong by not simply applying strict liability. This is an easy case for res ipsa loquitur.]

Reciprocity rationale of torts: Victim has a right to recover for injuries caused by a risk greater than in degree and different in order from those created by the victim and imposed on the D.

To find optimal level of care consider:1. error cost2. admistrability3. incentive effects

NuisanceLand can’t move. It can’t get out of the way of the nuisance.

Private Nuisance: a condition or activity that interferes with the possessor’s use and enjoyment of his land by incorporeal or non-trespassory invasions to such an extent that the landowner cannot reasonably be expected to bear without compensation.

A. Private nuisance generally involves incompatible uses of two parcels of land.B. The issue is not D’s conduct but the type of harm suffered by P.

56

Page 57: Torts - University of Chicagoblsa.uchicago.edu/first year/TORTS/Torts-Epstein2002b.doc · Web viewTorts Introduction: Three categories of torts Intentional torts: where the defendant

Torts

a. Large element of live and let live for low-severity, high-frequency, reciprocal nuisance

b. As long as the primary goal is not malice: spite fence (but can’t have any practical use)

C. Measure is the ordinary person-D. One way to prove nuisance is a reduction in the market value of land.E. Examples:

a. Pollution of air, land, water by dust or smoke, odors, chemicals, noise.b. Heavy traffic or intensen light shone directly into P’s house might countc. Electromagnetic radiation or stray voltage might be a nuisance if it is a tort at all

i. Vogel v Grant-Lafayette Electric Cooperative (WI 1996): stray voltage released by D’s system harms P’s cows. D promptly remedied problem when it was told about it. Held: stray voltage can be nuisance b/c interfered with use of land. But it was an unintentional nuisance.

F. Trespass versus nuisancea. No tolerance for trespass regardless of whether there was a harmb. High tolerance for nuisance—P has to show actual and unreasonable harm

i. Harm caused by a predecessor in title to the land is not a nuisanceELEMENTS:

1. P has to have a possessory interest in the land.2. It must be substantial enough to get beyond live and let live

a. Unreasonableness measured by the reasonable expectations of the normal person occupying P’s land.

i. Higher standard of living, lower tolerance for nuisanceb. Factors to consider:

i. Non-reciprocal: one harm is much greater and outside ordinary useii. Character of the neighborhood or social expectations

1. nuisance is location specificiii. Magnitude, frequency, or duration exceeding neighborhood normsiv. Priority in time or priority in space: coming to the nuisancev. Gravity of harm

vi. Lack of utility of D’s activity1. even if D’s activity is properly conducted in a proper location, if

it is of no social utility, it may still be a nuisance.vii. Utility of D’s activity

1. social value is a factor, but if harm outweighs the utility, cts will find a nuisnce, if

2. the activity would also count as a nuisance if its utility were disregarded

3. If social utility is high, P might reasonably be expected to put up with a higher level of nuisance

a. Nuisance is simultaneously a public good and a public bad, so you aggregate all the gains and losses to establish the benchmark of harm.

b. Madison v Ducktown Sulphur (TN 1904): the land is worthless except for mining which is worth a lot. Enjoining mining would mean killing the town and destroying $200k of property for land worth $1000.

R2T §826 Unreasonableness of Intentional InvasionAn intentional invasion of another’s interest in the use and enjoyment of land is unreasonable if

57

Page 58: Torts - University of Chicagoblsa.uchicago.edu/first year/TORTS/Torts-Epstein2002b.doc · Web viewTorts Introduction: Three categories of torts Intentional torts: where the defendant

Torts

a) the gravity of the harm outweighs the utility of the actor’s conduct, orb) the harm caused by the conduct is serious and the financial burden of compensating for this and similar

harm to others would not make the continuation of the conduct not feasible.R2T §821F: There is liability for a nuisance only to those to whom it causes significant harm, of a kind that would be suffered by a normal person in the community or by property in normal condition and used for normal purpose.

3. Coming to the nuisancea. Priority in time: prior use has stamped a neighborhood’s character, P cannot

expect to change that. (Assumption of risk)b. Or, D has to give way before changing use of P’s land and public good

i. P should be able to make use of his landii. back-end of a state enforced exchange imposed when D first makes use

of his property: P doesn’t sue D to enjoin the nuisance today, and D doesn’t plead statute of limitations or assumption of risk tomorrow.

c. Ensign v Wells (MI 1948)-kennel4. D has to act

a. D is usually not responsible for the natural condition of his land or natural animals or vegetation on it.

i. Unless D knows of the hazard and responds imperfectly or not at allii. Merriam v McConnell (Il App 1961): bugs infesting P’s trees not a nuisance b/c D

didn’t put them there.iii. Puritan Holding Co v Holloschitz (NY Sup Ct 1975): abandoning building in area

where housing prices are increasing.b. D’s act must be both intentional and unreasonable

i. Even if the conduct is beneficial, if it is deliberate and persistent it might become per se unreasonable, especially if D continues it after knowing it is causing a nuisance to P.

ii. Certain activities are nuisances per se, regardless of the care with which they are done, regardless of where and when they are undertaken

1. ultrahazardous activities2. continuous or repeatedly discharging substances depriving P of

her rights to enjoyment, no matter if D was not negligent.iii. Strict liability approach: All substantial physical harms to person or

property are actionable regardless of benefit to public.5. Causation: still have to show cause in fact and proximate cause, with natural conditions

raising the greatest hurdles with respect to proximate cause.6. Affirmative defenses:

a. P is passive—so little room for contributory negligence b. No extrasensitivity defense—work this out in contract/landlord-tenant lease

i. Will only assign liability if act harms persons of ordinary sensibilitiesii. Rodgers v Elliot (MA 1888): bell ringing—long custom. P suffered sunstroke, had

convulsions whenever D rang nearby churchbell. D refused to stop. No nuisance b/c the standard by which one can use his property without it becoming a nuisance to others is an objective one. It does not have to take into account people with special sensitivities and should not be measured by people long-accustomed to the nuisance.

1. long-custom creates a sort of prescriptive easement of which world has notice

7. Non-invasive nuisances: light or air blocked, aesthetically repugnant activitiesa. Allowing non-invasive nuisance restricts possible uses of land too much.

58

Page 59: Torts - University of Chicagoblsa.uchicago.edu/first year/TORTS/Torts-Epstein2002b.doc · Web viewTorts Introduction: Three categories of torts Intentional torts: where the defendant

Torts

b. Blocking air, light, view—no action except in case of pure spite fencei. Fontainebleau Hotel v 45 25 (FL App 1959): Rule: one must use his property so as not

to injure he lawful rights of another. There is no lawful right to a prescriptive easement in light and air.

ii. But, Prah v Maretti (WI 1982): unreasonable obstruction of access to sunlight might be a private nuisance b/c of societal use for alternative energy.

c. The law of nuisance is not concerned with aesthetics, so having ugly property which drives down the value of surrounding property is not sufficient.

i. Junkyard on lawn: if it’s really out of character with neighborhood, ct may find nuisance

d. Anticipated harms—cts differ, but usually will not find nuisance until harm becomes actual.

8. Remediesa. Damages

i. Compensatory1. periodic damages allows more accurate assessment of damages,

but higher adminstration costs.2. Lump sum payment for loss—inaccurate assessment, lower costs.3. Boomer v Atlantic Cement (NY 1970): cement factory pollutes air, but how

do you assess damages? Ct rejects injunction or temporary injunction pending solving the problem b/c technology doesn’t allow solution and we don’t want to shut factory down. Prefers forcing plant to pay one-time permanent damages. Dissent: prefer temporary injunction pending solution.

ii. Punitive damages inegregious casesb. Injunction

i. Abating and modifying the nuisance ii. Compensated injunction: abates nuisance but P has to pay the costs of

the abatement—only one case:1. Spur Industries v Del E Webb Development (AZ 1972): developer built

sub-division next to feedlot, had to pay feedlot owner to move b/c owner had originally built very far from town.

iii. Issue to consider in issuing a permanent injunction:1. Serious risk of D’s insolvency2. Protecting other innocent 3rd parties not joined to suit3. Lower administrative costs b/c court doesn’t have to assess

damage amountsc. P is has duty to mitigate under any remedy scheme

Public nuisanceRule: General damages from public nuisance are controlled only by direct public action, usually administrative regulation or criminal prosecution. The private action is maintainable only for special, peculiar, or disproportionate harm to the individual P.

Rationale: Administrability: otherwise too many small suitsTwo classes of special damages:

1. total loss of access to private landa. blocked road, polluted waterb. partial loss and/or inconvenience not sufficient

2. personal injury that is different in kind from injuries suffered by general public

59

Page 60: Torts - University of Chicagoblsa.uchicago.edu/first year/TORTS/Torts-Epstein2002b.doc · Web viewTorts Introduction: Three categories of torts Intentional torts: where the defendant

Torts

a. example: D’s pollution causes everyone respiratory problems, and P’s problem is the same, P has no special private cause of action.

Problem: assessing who has standing to bring private suit.Just give right of action to inner circle—people directly harmed and if that provides an

adequate deterrence, stop there.Union Oil v Oppen [P] (9th Cir 1974): fisherman sue for oil spillage that kills fish and ruins their business. Issue is who is best cost-avoider, which turns out to be the oil company that caused the damage.

Remedy scheme:Use a system of direct regulation

1. tax (like damages)2. regulation (like injunction)

Prefer strategies that 1. don’t have to be constantly readjusted2. where individuals don’t have to cooperate with each other

Products LiabilityThree classes of products liability:

1. Manufacturing defects—strict liability2. Design defects—negligence3. Warnings—negligence

a. Issue is design defects and warnings: how to set standards on what is reasonably safe

b. Most modern products liability cases fall into the last two categories4. • Key: D not in possession of product when it causes harm—what is the duty of care?

• The purpose of the products liability system ought to be deterrence—what kind of liability will get to this goal?• Products are not homogeneous, which makes it impossible to find a single rule that will cover all products indiscriminately.

Products have different lifespans They move through the stream of commerce in different ways They all have different chains of distribution Some are used by both highly trained and untrained people

DevelopmentA. PHASE ONE: privity limitations prevented the injured party from suing the remote supplier

of the product.1) Rule: the liability of the contractor or manufacturer for negligence in the

construction or sale of articles which he makes or vends is limited to the persons to whom he is liable under his contracts of construction or sale.

a) Winterbottom v Wright (Ex 1842): D contracts with Postmaster to supply and maintain coaches for mail delivery. 3rd party contracts to supply horses and drivers. P is a driver, injured b/c of defective coach. Held for D b/c P not in privity of contract with D, so there is no cause of action. Rationale: allowing such an action would make D liable for actions by any user of the coach no matter how remote.

b) Three exceptions in American cases : (Huset)

60

Page 61: Torts - University of Chicagoblsa.uchicago.edu/first year/TORTS/Torts-Epstein2002b.doc · Web viewTorts Introduction: Three categories of torts Intentional torts: where the defendant

Torts

1. When negligence of manufacturer or vendor is imminently dangerous to life and committed in preparation of product intended to preserve, destroy, or affect life. (mislabeled poison)

2. owner invites user to use defective product on owner’s premises and user is injured, owner may be liable. (badly built scaffold)

3. one who sells or delivers an article which he knows o be imminently dangerous to life or limb to another without notice of its dangerousness, is liable for any injury that might have been reasonably anticipated, with or without contractual relationship. (knowledge of defect, fraud, misrepresentation)

i. Huset v J.I. Case Threshing Machine (8th Cir 1903): P injured by defective threshing machine which D sold knowing that one of the parts would not serve the purpose for which it was meant.

ii. Kuelling v Roderick Lean Manufacturing Co (NY 1905): P injured by defective roller D made out of bad wood and with a knot that D had tried to hide by putty and paint. Action allowed as fraud.

B. PHASE TWO: reject privity limitation and impose liability in negligence on a remote seller.1) Rule:

1. If a product is reasonably certain to cause harm if defective, it is a dangerous thing.

2. Manufacturer of dangerous thing is liable when:a. it is foreseeable that it will be used by those other than the purchaser without

undergoing further testsb. the danger inherent in a defective thing is probable, not merely

possiblea) MacPherson v Buick Motor Co. (NY 1916) Cardozo, J. P=car dealer injured

when defective wooden wheel of new car crumbled. Issue: does manufacturer own duty of care to anyone but immediate purchaser. Held: in this case, the one person not likely to use the car is the dealer. Silly to prevent buyer from having cause of action. Maker of finished product has duty to test parts made by sub-manufacturers. Rule: if manufacturer is negligent where danger is to be foreseen, he is liable even to users other than the one with whom he is in privity of K. [Cardozo combined exceptions 1 and 3 and used negligence standard, turning exception into the rule.]

C. PHASE THREE: strict liability should govern manufacturer’s liability.1) Rule—R2T §402A:2) Rationale:

1. Manufacturer is in best position to minimize losses arising from use of its product

a. Discourage sale of defective products2. Manufacturer can spread risk through insurance and price increase3. Eliminate proof requirement, which is hard for P to produce4. Products are analogous to prepared food, for which there is criminal

liability5. Corrective justice: why should victim have to pay when injured through no

fault of his owna. Commentary : unlike abnormally dangerous activities, in which D is in

possession of the dangerous instrumentality, in products liability suits, D is never in possession when the harm occurs. Privity limitations = defensible means for

61

Page 62: Torts - University of Chicagoblsa.uchicago.edu/first year/TORTS/Torts-Epstein2002b.doc · Web viewTorts Introduction: Three categories of torts Intentional torts: where the defendant

Torts

liability to track possession, except when party out of possession is in better position to avoid theloss.

6. Escola v. Coca-Cola Bottling Co (CA 1944), Traynor, J.: P injured when Coke bottle exploded in her hand. P = waitress, so not in privity of K with bottler. Held: manufacturer is absolutely liable b/c he is the best risk avoider, the best loss-spreader, it is in public interest to discourage manufacturer’s from marketing defective products, too difficult for P to prove negligence, the issue is not a K warranty issue but a tort issue, so privity is not necessary.

a. Traynor’s two limitationsa. Defect narrowly defined as latent miscarriage in the manufacturing

processb. Normal and proper use by P where the defect is traceable to the product in

the condition in which it reached the market.b. Traynor assumes market can bear the costs of strict liability—Epstein thinks it is

better to spend the money on first party insurance.3) Contract Approach—Implied warranties, actions against retailer.

1. Injured P brought action against retailer for injuries caused by defective products

a. Privity couldn’t bar P’s cause of action2. Two regimes of K with sequential use of product where multiple parties

take products from each other in successiona. Loss minimalization: have a sequential K reflecting parties’ view of

ideal distribution of risksb. Unconscionability: treat warranties with suspicion.

3. McCabe v L.K. Liggett Drug Co (MA 1953): P’s agent bought coffeemaker from D. It was delivered, sealed, to P who used it in the proper manner. She noticed it was defective, kept using it, after several times, it exploded, injuring her. Sale of product carried with it implied warranty of merchantable quality. That machine exploded, when properly used, is res ipsa loquitur proof that it was defective. If it was defective, it was not of merchantable quality.

a. Merchantable quality : goods are reasonably suitable for the ordinary uses for which goods of that description are sold.

b. First question when there is a product failure: is it a common mode failure (a manufacturing or quality control defect) or a unique failure.

a. If the former probably the whole lot was defective, so be suspicious of a single bad product

b. If the latter look to P’s mistake or local environment4. Foodstuffs cases

a. K approach creates anomaly in foodstuffs cases b/c goods move rapidly from hand to hand

b. Manufacturer is happy to warranty product as pure b/c the problem of skilled v. careless users doesn’t come up.

c. But, early cases refused to extend cause of action to 3rd parties who received the goods from the buyer

a. Example: Chysky v Drake Bros (NY 1923): waitress given D’s cake, bites into it and is cut by nail in the cake. No action b/c statute only covers implied or express warranty to buyer, so there was no contractual relationship with P.

d. Next step: rejects Chysky, extends cause of action to buyer and associate against retailer

62

Page 63: Torts - University of Chicagoblsa.uchicago.edu/first year/TORTS/Torts-Epstein2002b.doc · Web viewTorts Introduction: Three categories of torts Intentional torts: where the defendant

Torts

a. Example: Greenberg v Lorenz (NY 1961): P given canned salmon by her father who had bought it from D a retailer. Salmon contained metal slivers and P injured. Court overturns Chysky and allows cause of action.

e. UCC 2-318 governs scope of warranty in regard to third party beneficiaries, applies to all products, offers three alternatives

a. A covers any natural person who is part of family or household of buyer or guest in his home, seller can’t disclaim or limit warranty

b. B covers any natural person who is a foreseeable user who is injured in person. Seller can’t disclaim or limit.

c. C covers any person who is foreseeable user injured by breach. A seller may not exclude or limit warranty with respect to injury to the person of an individual to whom the warranty extends.

f. Defective products and implied warrantiesg. Henningsen v Bloomfield Motors (NJ 1960): Henningsen buys new car from D,

intending it as a gift to his wife, P. P injured by defect in car. By limited warranty in K, P can’t sue for consequential damages b/c not in privity. Judgment moves products liability from private K issue, by invalidating limited warranties, to tort issue, allowing person not in privity to sue.

h. Epstein on limited warranties: sorts out the better manufacturers form the worse ones b/c honoring the warranty costs $, so manufacturer will try to build better product.

i. Greenman v Ybua Power Products (CA 1963): man injured by defective lathe that manufacturer’s brochure said was safe. Rule: a manufacturer is strictly liable when an article he places on the market, knowing that it is to be used without inspection for defects, proves to have a defect that causes injury to human beings. Held: don’t do this in K with warranties and privity requirement, but rather in tort on strict liability basis. Rationale: put costs on manufacturer rather than victim. Warranties = inadequate. “Implicit in the machine’s presence on the market was a representation that it would safely do the jobs for which it was built.” Limit: had to be correct and normal use.

D. RST §402A: Special liability of seller of product for physical harm to user or consumer.(Strong defendants’ document)

1) One who sells any product in a defective condition unreasonably dangerous to the user or consumer or to his property is subject to liability for physical harm thereby caused to the ultimate user or consumer or to his property if;

a. The seller is engaged in the business of selling such a product, andi. Business of selling: applies to manufacturer, distributor, retailer,

wholesaler, restauranteur. Seller doesn’t only have to sell such products. Doesn’t apply to occasional seller not engaged in selling the product as part of his business.

b. It is expected to and does reach the user or consumer without substantial change in the condition in which it is sold

2) The rule stated in (1) applies althougha. The seller has exercised all possible care in the preparation and sale

of his product, andb. The user or consumer has not bought the product from or entered into

any contractual relation with the seller.Caveat: no opinion expressed whether the rules may not apply to harm:

1) to persons other than users or consumers, 2) to the seller of a product expected to be processed or otherwise

substantially changed before it reaches the user or consumer, 3) to the seller of a component part of a product to be assembled.

63

Page 64: Torts - University of Chicagoblsa.uchicago.edu/first year/TORTS/Torts-Epstein2002b.doc · Web viewTorts Introduction: Three categories of torts Intentional torts: where the defendant

Torts

A defect is that which is not what the consumer expected. Defective condition: rule applies only where product is

already defective when it leaves seller’s hands, and when the ultimate consumer doesn’t expect that, and where defect will be unreasonably dangerous to

consumer. 1. Unreasonably dangerous: dangerous to the extent

beyond that which would be contemplated by the ordinary consumer who purchases it, with the ordinary knowledge common to the community as to its characteristics. Not something dangerous in excess or in an abnormal use.

2. Seller may be required to give directions or warning to prevent product from being unreasonably dangerous. Seller allowed to assume that warning will be read and heeded.

3. Seller who supplies inherently dangerous but useful product (e.g. medication) is not held strictly liable for harms as long as proper warnings given, proper packaging and marketing.

Seller not liable for defect that occurs in subsequent mishandling. Burden of proof of defect is on P

Seller not liable when consumer uses product in abnormal way

No distinction between problem with the product or with its packaging

Warranties not the basis for the cause of action. Contributory negligence doesn’t apply to failure to

discover defect or guard against it, but does apply to voluntary assumption of risk if user discovers defect and uses product anyway.

~~R2T is a defendant’s document on product liability~~E. Phase Four: Modern product liability—defective design and duty to warn are the

primary issues1) What are products: tangible personal property, but not blood and tissues; in some

cases real property and electricity2) Distinction between services and products

1. Products fall under strict liability, services under negligence.a. Negligence covers service b/c the success or failure of the service

performed depends not only on competence of D but also on thing or person on which service is being performed—negligence standard prevents D from having to insure the thing/person being worked on.

b. Products fall under strict liability insofar as D had complete control over them and is in a position to avoid risk

c. Problem arises in hybrid situations such as a pharmacist filling a prescription.

d. Cafazzo v Central Medical Health Services (PA 1995)—P wants to sue Dr. for defective mandibular implant. Held: Dr was only performing a service and is not strictly liable for a product defect.

3) Potential Ds—all sellers from manufacturer on down—all held to same standard, all held to same liability as manufacturer—even when intermediaries have been entirely passive.

64

Page 65: Torts - University of Chicagoblsa.uchicago.edu/first year/TORTS/Torts-Epstein2002b.doc · Web viewTorts Introduction: Three categories of torts Intentional torts: where the defendant

Torts

1. Exception: casual sellers & dealers in used products.a. Dealer in used products is liable for any defects it introduces in

remanufacturing the product4) Potential Ps—all users and consumers of products

1. Modern law also allows bystanders to recover from manufacturera. Rationale: bystander harmed by process which he did not make

5) What harms are covered: physical harms but not economic harms1. line between them is difficult to draw2. Economic losses: disappointed economic expectations—controlled by K,

essentially the product damages itselfa. Tort losses: harm above and beyond disappointed expectationsb. Casa Clara Condo Assoc v Charley Toppino & Sons (FL 1993): homeowners

can’t sue in tort maker of defective concrete used in building their houses b/c this was an economic harm. No “other product” involved, just the house as a whole. Homeowners not in strict privity with concrete supplier.

6) Rule—R3T:PL §§1-2.§1: Liability of commercial seller or distributor for harm caused by defective productsOne engaged in the business of selling or otherwise distributing products who sells or distributes a defective product is subject to liability for harm to persons or property caused by the defect.

§2: Categories of product defects [for purposes of determining liability under §1]A product is defective when, at the time of sale or distribution, it contains a manufacturing defect, is defective in design, or is defective b/c of inadequate instructions or warning. A product:a) contains a manufacturing defect when the product departs from its intended

design even though all possible care was exercised in the preparation and marketing of the product;

b) is defective in design when the foreseeable risks of harm posed by the product could have been reduced or avoided by the adoption of a reasonable alternative design by the seller or other distributor, or a predecessor in the commercial chain of distribution, and the omission of the alternative design renders the product not reasonably safe;

c) is defective b/c of inadequate instructions or warnings when the foreseeable risks of harm posed by the product could have been reduced or avoided by the provision of reasonable instructions or warning by the seller or other distributor, or a predecessor in the commercial chain of distribution, and the omission of the instructions or warnings renders the product not reasonably safe.

Classification scheme: construction, design, and warning defects Keeps strict liability for manufacturing defects, but imposes limited

more obligation to make product designs, warning, and instructions ‘reasonably safe’ Continues to exclude casual sellers

Stage one of inquiry into product defectsa) identify class of product defectsb) identify standard of liability for each defect

65

Page 66: Torts - University of Chicagoblsa.uchicago.edu/first year/TORTS/Torts-Epstein2002b.doc · Web viewTorts Introduction: Three categories of torts Intentional torts: where the defendant

Torts

66

Page 67: Torts - University of Chicagoblsa.uchicago.edu/first year/TORTS/Torts-Epstein2002b.doc · Web viewTorts Introduction: Three categories of torts Intentional torts: where the defendant

Torts

Product Defects:Three categories1. Manufacturing defects—product is not made in accordance with its own specifications2. Design defects3. Warning defects

Manufacturing Defects RULE: manufacturer’s liability for a defective product is predicated upon negligence in the

manufacture or design of the product. P has to prove that injury resulted from a condition of the product which was unreasonably dangerous and which existed at the time the product left the manufacturer’s control.

Defect doesn’t have to manifest itself at once Law doesn’t set up independent standards, but just uses internal standards of manufacturer. Have to show that defect renders product dangerous

Pouncey v Ford Motor Co (5th Cir 1972)—radiator fan broke while P worked on (used) car, injuring him. Battle of the experts. Verdict for P. D claims that evidence couldn’t support verdict, but held jury could infer negligence from surrounding evidence. Complication: food cases where some natural ingredient is both natural and dangerous,

Example: fish bone in fish chowder Test becomes consumer expectations: natural ingredient is defective if not expected.

P can reach jury now even without showing what was defective, based on a variant of res ipsa loquitur, if P can show that his actions/use did not make the product dangerous.

Welge v Planters Lifesaver (7th Cir 1994): peanut butter jar collapsed as P was closing it. No evidence of improper use/ storage, but also couldn’t point to defect that caused collapse. P allowed to go to jury. R3T §3 allows P to recover without specifying defect responsible for the harm if other causes of the harm can be excluded.

R2T and R3T impose strict liability “when the product departs from its intended design even though all possible care was exercised in the preparation and marketing of the product.” Rationale for strict liability: manufacturer controls whole production process, don’t use up resources examining each part of the process in litigation, measure it by the output. This lets manufacturer design maximal inputs to ensure safe products.

Design Defects Concealed design defects are treated like manufacturing defects—require manufacturer to make his product

design meet his own performance standard. You can never have a design defect case where you can’t point to what’s wrong b/c with design issues you have perfect knowledge. If it is unclear what the defect is—try it under construction defect.

Crashworthiness cases expanded design defects liability Defect lay in car’s failure to protect occupant [P] against crashed caused by P or by 3rd party Decided on negligence

It doesn’t make sense to hold D strictly liable when D is being required to protect P from a 3rd person. Use Hand formula: are the burdens of precautions worth taking in light of the risk to be averted?

Base expectations of safe operating on ‘intended use’ of the car—including foreseeably risk of accidents. Volkswagon of America v Young (MD 1974): P killed in second collision when 3rd party hit his car and

he was thrown b/c seat brackets broke. Issue: does crashworthiness fit into intended use of a car? Rule: under negligence, manufacturer is liable for a defect in design which it could reasonably have foreseen would cause or enhance injuries on impact, which are not obvious to user, and which in fact lead to injuries in a collision. No recovery if danger is obvious to user. Safety features only have to be reasonable—don’t have to protect against every contingency.

67

Page 68: Torts - University of Chicagoblsa.uchicago.edu/first year/TORTS/Torts-Epstein2002b.doc · Web viewTorts Introduction: Three categories of torts Intentional torts: where the defendant

Torts

Standards of design defect liability—theories Open and obvious dangers —offer and acceptance model. Shifts blame to consumer.

once P has manifest notice of the danger, he can decide whether to use the product or seek out a safer one.

Through the 1950s—assumption of risk assigned to P when danger was obvious. Campo v Scofield (NY 1950)—P’s hands injured when caught in onion topping machine. Recovery denied b/c user could see the danger and manufacturer not obligated to make the machine accident proof or foolproof. Rule: manufacturer is under no duty to guard against injury from a patent peril or from a source manifestly dangerous. But, Micallef v Miehle Co (NY 1976)—lack of safety mechanism on printing machine. Held: machine design could be defective even if danger was obvious b/c manufacturer in best position to improve the design and liability gives him incentive. Rule: manufacturer is obligated to exercise that degree of care in his design se as to avoid any unreasonable risk of harm to anyone who is likely to be exposed to the danger when the product is used in the manner for which it was intended as well as unintended yet reasonably foreseeable use. Reasonableness = balance of cost, likelihood of harm, gravity of harm, function.

Rationale: simplified litigation, clearly delineates spheres of responsibility between manufacturer and consumer, D doesn’t have to justify design to jury.

But jury will still find for P when a simple safety feature could have prevented injury. Consumer expectations —totality of the consumer’s knowledge about product

Looks to appearance, common knowledge about product’s use and limitations, literature that accompanies it.

Tries to determine if design reflects its safe and intended use—any other use is at consumer’s risk Example: a small convertible sports car’s design is not communicating that it is intended to be safe

in a roll-over crash. R2T §402A seems to use this test “defective condition unreasonably dangerous” taken to mean that

article must be dangerous to an extent beyond that which would be contemplated by ordinary consumer with ordinary knowledge of how it works. Example: everyone knows cigarettes are dangerous, butter causes high cholesterol. Linegar v Armour of America (8th cir 1990): P killed when bullet entered chest under his arm where bullet-proof vest did not cover. No recovery allowed b/c the limited coverage of the vest was patent. The choice in vest style was based on a trade-off with comfort. Law doesn’t obligate manufacturers only to make the safest possible design, b/c other factors like cost and consumer preference are involved.

Problem: not always clear what is safe and intended use based on the design. Statutory standards —rely on regulation to set safety standards

Products complying with safety standards would be treated as safe. Works best for products subject to comprehensive regulations (e.g. cars, planes) But, cases have held that standards may be result of interest group pressure, so they are taken to

establish only legal minimums. Use basic negligence rules instead where noncompliance with standard is conclusive evidence of a defect. But compliance is only evidence of safe product design—might establish presumption of safety but doesn’t grant immunity from suit unless legislature has addressed the issue specifically. Standards can lag behind technology b/c of rigidities in bureaucratic system

State of the art —products are reasonably safe when D’s product design conforms to state of the art of the relevant trade or industry. D’s defense could be

Design conforms to industry custom—carries the least weight, b/c P could always point out that other designs are safer.

Design reflects the safest and most advanced technology in use Design reflects cutting edge knowledge

State of the art will only rule out design defect liability is the product was manufactured before the new technology was created—which of course no jury would expect anyway.

Law follows TJ Hooper rule: an entire industry could lag behind the technology, but that doesn’t make it excusable.

68

Page 69: Torts - University of Chicagoblsa.uchicago.edu/first year/TORTS/Torts-Epstein2002b.doc · Web viewTorts Introduction: Three categories of torts Intentional torts: where the defendant

Torts

So far, the theories have mostly been rejected, leaving: Reasonable alternative designs —negligence standard generally used.

R3T PL §2(b): A product is defective in design when the foreseeable risks of harm posed by the product could have been reduced or avoided by the adoption of a reasonable alternative design by the seller and the omission of the alternative design renders the product not reasonably safe. Courts are divided over the standard. Doesn’t require alternative design be one in general use—relies on experts to establish safe alternative. Turns on a cost-benefit analysis Barker v Lull (CA 1978): high lift loader incorrectly used by inexperience driver. [Should have been settled as a workers comp case.] P claims there should have been more safety features. D says those features would not have been safer and that they are not industry standard. P only has to show that some condition made the product unsafe, but does not have to show a specific defect. Held for P b/c manufacturer has to foresee reasonable misuse.

Problem: A product that is made safe for the untrained or negligent user might end up being more dangerous for the knowledgeable or proper user—then manufacturers are in real trouble.

Cf. Risk-utility test—makes everything relevant and nothing dispositive—favors P. Usefulness and desirability of the product—its utility to the user and to the general public The safety aspects of the product—the likelihood it will cause injury and the probably seriousness of the injury Availability of alternative product which would meet the same need and not be as unsafe Manufacturer’s ability to eliminate the unsafe character of the product without impairing its usefulness or making it too expensive User’s ability to avoid danger by the exercise of care in the use of the product User’s anticipated awareness of the dangers inherent in the product and their avoidability based on general public knowledge and accompanying information Feasibility, on the part of the manufacturer, of spreading the loss by setting the price of the product or carrying liability insurance.

Mixes up design defect, P’s conduct, whether product should be on market at all, steps by manufacturer to make it safer, cost, insurance.

Every form of evidence is admissible O’Brien v Muskin Corp (NJ 1983): P injured diving into 3 feet of water in above ground plastic pool. Claimed vinyl covering on bottom was defective b/c too slippery, causing P’s arms to slip and him to hit his head. D argued that this covering was the only one possible for the product.

Product is per se defective b/c it is dangerous. Potter v Chicago Pneumatic Tool (CN 1997): P claims damages from excessive vibration caused by grinding tools over 25 years, claiming tools were defectively designed without safety features. Assessed on risk-utility test. Ct ignores evidence that the product didn’t comply with industry standards, and Ct ignores downstream alterations by users.

Problems—policy issues : Present system of design defects requires manufacturer’s to guard against many forms of user neglect, deliberate misconduct, or natural catastrophe as long as these are foreseeable Suits try hypotheticals b/c P has to reverse engineer the product to establish how harm could have been prevented System is unnecessarily expensive and unpredictable

No evidence that litigation leads to safer designs. Designs have to be set from ex ante perspective, and tradeoffs made.

Law doesn’t take into account cost of the design features—if products cost too much, people won’t buy them and will continue to use older, more dangerous products.

69

Page 70: Torts - University of Chicagoblsa.uchicago.edu/first year/TORTS/Torts-Epstein2002b.doc · Web viewTorts Introduction: Three categories of torts Intentional torts: where the defendant

Torts

Duty to Warn and Instructions—what is sufficient to protect user or consumer’s sovereignty of choice?Step one: don’t misrepresentStep two: provide label with minimum information—forces consumers to get info for

themselves (Epstein’s favorite stage)Step three: provide all possible warnings.

Relation to design defects RULE: as long as D has no duty to design out obvious features of his product, then he is under no duty to warn of them. Warnings and design features = substitutes for each other

Manufacturer will make designs safer rather than use warnings on apparently safe products with latent dangers But some products need warnings to supplement design features

R2T rule = too broad: where a warning is given, the seller may reasonably assume that it will be read and heeded.

Drugs and chemicals: deadly but useful poisons, drugs beneficial in isolation but deadly in combination with others, products safe when properly stored but otherwise dangerous. Bans are inappropriate so warnings and instructions are used to control risk.

Reasonableness standard for warnings: R3T PL §2: A product…(c) is defective b/c of inadequate instructions or warnings when the foreseeable risks of harm posed by the product could have been reduced by the provision of reasonable instructions or warnings by the seller or a predecessor in the commercial chain of distribution and the omission of the instructions or warnings renders the product not reasonably safe. Scope of the Duty —who should warn whom?

Manufacturer of finished product has to create warning and all downstream distributors have to transmit it with the product.

Manufacturers of component parts upstream are insulated from the requirement. Retailer is liable when manufacturer fails to include warning in a sealed package

Why impose this on retailers who would have to open the package to discover the defect? Retailer is ill-suited to avoid the risk, whereas consumer will discover warnings are missing.

Problem: when manufacturer markets product directly to consumer. MacDonald v Ortho Pharmeceutical: since contraceptive was foreseeably used with minimal instruction from doctor, manufacturer had to be more explicit in warning to user. R3T PL §6 Presciption drugs and medical devices:

d) A prescription drug is not reasonably safe due to inadequate instructions or warnings if reasonable instructions or warnings regarding foreseeable risks of harm are not provided to:

(A) prescribing and other health-care providers who are in a position to reduce the risks of harm in accordance with the instructions or warnings, or

(B) the patient when the manufacturer knows or has reason to know that health-care providers will not be in a position to reduce the risks of harm in accordance with the instructions or warnings.

RTT uses “reasonable” standard to keep warnings out of strict liability realm. Scope of the Duty : How?

Instructions/warnings have to be transmitted in a way calculated to reach the consumer in its ordinary use of the product

70

Page 71: Torts - University of Chicagoblsa.uchicago.edu/first year/TORTS/Torts-Epstein2002b.doc · Web viewTorts Introduction: Three categories of torts Intentional torts: where the defendant

Torts

Broad range of distribution patterns makes this complex Usual RULE: reasonable efforts should be made to bring the risk to the attention of the actual users, or, if that is not possible, to qualified intermediaries who then can either act on the information required or pass it on to those who need it.

Example: drug manufacturer warns doctor who warns patient Mass vaccination: Where downstream product control is extensive, any duty to warn should attach not to the manufacturer but to the administrator of the program.

Davis v. Wyeth Laboratories (9th Cir 1968): mass polio vaccination program, not run by doctor, aided by vaccination producer, there was a duty to warn of risks of using the live polio vaccine, especially b/c risk, though small, would have a very significant effect.

What are adequate warnings ? Identify the risks for which warnings are required

Where there are serious risks, seller is more likely to know about them than user Where risks are minor, seller is not required to produce information/warnings Hard cases: intermediate levels of dangers

E.g. allergies known to affect a small fraction of the product’s users. Balancing test:

Frequency of harm Severity of harm warning needs to be in lay terms people can understand—MacDonald v

Ortho Pharmeceutical: “the adequacy of such warnings is measured not only by what is stated, but also the manner in which it is stated.” E.g. has to be appropriate level of urgency.

Duty to warn also requires manufacturers and sellers to take reasonable care to test products to discover risks for which warnings are appropriate.

Sellers can’t be expected to pass warnings to users: Product seller doesn’t have and cannot at reasonable cost acquire the information on which warnings might be based

But strict liability seems to require warnings of unknowable dangers anyway

R3T requires P to establish that seller/manufacturer could have known of the danger in the first place

Dangerous condition is open and obvious—so why spend money on warnings But condition may be obvious while danger is not

What kinds of warnings are required to convey information about a known or knowable risk

Seller has to supply warning that allows for informed use, or nonuse, in light of serious dangers.

But what this means in practice is unclear—how explicit and detailed the warning has to be of every possible danger. P has burden of showing that an additional warning would have

prevented or reduced the injury. Problem: too many warnings of even trivial risks scare users away from

products that are genuinely useful and needed. “Failure-to-warn cases have the curious property that, when the episode is

examined in hindsight, it appears as though addition of warnings keyed to a particular accident would be virtually cost free….The primary cost is, in fact, the increased time and effort required for the user to grasp the message. The inclusion

71

Page 72: Torts - University of Chicagoblsa.uchicago.edu/first year/TORTS/Torts-Epstein2002b.doc · Web viewTorts Introduction: Three categories of torts Intentional torts: where the defendant

Torts

of each extra item dilutes the punch of every other item.” Cotton v Buckeye Gas Products (DC Cir 1988): P sued D propane canister supplier for not giving better warnings about dangers of explosive properties of propane and proper storage—even though P’s employer warned him about proper handling.

Problem: holding manufacturers liable for every trivial risk about which they do not warn drives up prices of useful and needed products like vaccines.

Solution: regulatory warnings made with the best current information that can be updated.

MacDonald v Ortho Pharmaceutical (MA 1985): P had stroke b/c she took birth control pills. Stroke not listed as one of the possible side effects, even though risk of blood clots in the brain was. Held: jury could reasonably find that warning was not explicit enough. Dissent: it was doctor’s responsibility to warn P and manufacturer adequately warned doctor.

When to warn? Usually transmitted at time of sale

But for durable goods, defect might only become apparent later Diversity of products creates different warning requirements for defects

arising after use has begun, e.g. airplanes issuing periodic advisories. R3T PL §10: limited obligation to warn after purchase

Duty confined to traps in the initial product Manufacturer is not required to inform consumers of new improvements But there is a duty to warn when

Peril of substantial impact in the original design manifests itself only after use

Users can be identified reasonably Costs of issuing the new warning are not prohibitive

Hindsight bias: in retrospect it looks as though D should have known about danger, but that is not reasonable.

Foresight rule: can only expect manufacturers to guard against reasonably foreseeable dangers based on foreseeable product use, and to perform reasonable testing prior to marketing a product and to discover risks and risk-avoidance measures that such testing would reveal. A seller is charged with knowledge of what reasonable testing would reveal.

Plus there is a continuing duty to warn at least purchasers as new dangers become known Gets incentives right—no incentive if danger is unknowable.

Unavoidably dangerous products: only option is to warn Example: blood transfusions before tests for AIDS or Hepatitis—couldn’t hold blodd

banks or hospitals to strict liability since there was no technology for identifying the dangers.

Opposite view: By imposing on manufacturers the costs of failure to discover hazards, we create an incentive for them to invest more actively in safety research—Asbestos litigation: Bethesda v Johns-Manville (NJ 1982)

In Kuhnian view of world, it is unrealistic to assume that science is able to know what sort of research to do in order to foresee all possible or even probably risks.

If in product use there is a duty to warn, which there is, and warning was incomplete, P did not assume the risk b/c incompletely informed.

72

Page 73: Torts - University of Chicagoblsa.uchicago.edu/first year/TORTS/Torts-Epstein2002b.doc · Web viewTorts Introduction: Three categories of torts Intentional torts: where the defendant

Torts

Adequate defense to duty to warn: I did not warn you, but you got the warning from somewhere else. There are so many avenues of information now, this can be very hard for P to disprove.

CAUSATION Show causal link between D’s wrongful conduct and P’s injury

Cause-in-fact and proximate cause work the same as in other torts Proof of cause-in-fact connection depends on type of product and type of harm

Problem with showing chemicals causing rare diseases Vasallo v Baxter Healthcare Corp (MA 1998): silicon breast implant leaks, causes damage to D. D’s held liable in part b/c they knew of potential for leakage and did not warn either doctors or users.

Superseding cause in manufacturing defects D can argue that the negligence of 3rd parties or a natural event counts as superseding

cause which would have nullified any warning P has so altered the product that it no longer contains the same construction design as

the original, so the product that caused harm is not the one D supplied To determine if downstream change was a break in the causal chain:

If altered product resembles original, go back to manufacturer—but let M get out of liability by having drafted good instructions regarding what user should change Manufacturer comes on location and alters product on request of user—still

strict liability Changes made unilaterally and reasonably by user

If change reduces total risk—fine, since M will usually profit Hold M liable for some downstream change that make product more

dangerous—the case law is divided [Epstein thinks this is stupid—thinks there should be no liability]

Young v Aeroil Products (9th Cir 1957)—decedent crushed to death by portable elevator which his employer had changed by adding equipment, throwing balance out of line and causing it to topple. Held: the thing being used was not the thing sold. Brown v US Stove Co (NJ 1984)—P burned by portable stove which his employer had modified to burn much hotter than it was designed to. Held: modifications did exonerate original manufacturer. But, Soler v Castmaster (NJ 1984)—user took off one safety feature and added another on die casting machine, worker injured. Court let jury find that the design was defective b/c it lacked a safety gate, then held that ‘in applying strict liability in torts for design defects, manufacturers cannot escape liability on the grounds of misuse or abnormal use if the actual us proximate to the injury was objectively foreseeable.”

Assumption of risk: P continues using the product without making needed repairs even after danger becomes manifest

Once causation is shown many courts and R3T shift burden to D to identify factors that might have reduced its causal significance: improper use, downstream changes, contributory negligence, etc.

Example: car had been repaired by retailer before defect appears=downstream change. Always consider joint causation issues b/c products liability damage is always indirect. P always plays some role.

73

Page 74: Torts - University of Chicagoblsa.uchicago.edu/first year/TORTS/Torts-Epstein2002b.doc · Web viewTorts Introduction: Three categories of torts Intentional torts: where the defendant

Torts

Manufacturing defect liability is strict, so courts allow modified res ipsa loquitur even if P can’t identify the defect

Welge v Planters Lifesavers—collapse peanut butter bottle. Design defect : have to show both that the design was defective and what the proper design should have been Warnings and Instructions causal issues:

Was P’s injury unrelated to the product? Example: Did P catch polio from the vaccine or naturally?

Would the warning have changed P’s behavior? Problems with drug causation:

Line is unclear between knowable and unknowable risks Actual harm can be long in the future, so causation is difficult to establish

Epstein view : live and die by the FDA: if the company systematically suppressed info from the FDA, get them on fraud; otherwise don’t assign liability they couldn’t foresee. This is not the law.

Affirmative Defenses based on P’s conduct Current regime adopted by most courts and R3T favors comparative negligence rule R3T: PL §17: Apportionment of responsibility between or among P, sellers and distributors or defective products, and others.

(a) A P’s recovery of damages for harm caused by a product defect may be reduced if the conduct of P combines with the product defect to cause the harm and P’s conduct fails to conform to generally applicable rules establishing appropriate standards of care.

First it got really bad: downstream use, which used to be source of all wrong, is now considered a foreseeable hazard that manufacturer has to take into account.

It matters not that preparing a product for use by idiots might make it dangerous for the safe users, and that the safe users have to subsidize the bad users. Issues of intermediate maintenance have fallen out of consideration.

Reaction to excesses is Daly: Daly v General Motors (CA 1978): decedent killed when car he was driving hit divider and he was thrown from car. Sued D for defective door lock design. But, decedent was drunk, wasn’t wearing seatbelt, and didn’t have doors locked. Court holds that CA will switch to a comparative fault regime even though product defect is strict liability. This creates incentives on both sides, since D still can’t avoid liability if the defective product is at fault.

Assumption of risk defense given narrow construction Courts do not allow manufacturers to contract out of product liability

74

Page 75: Torts - University of Chicagoblsa.uchicago.edu/first year/TORTS/Torts-Epstein2002b.doc · Web viewTorts Introduction: Three categories of torts Intentional torts: where the defendant

Torts

Damages Damages are implicit in every case.

Damages establish the implicit price payable for a violation of a legal right. Damages set by the expected cost of D’s act, which depends on probability of D

being found liable and the damage awarded. Damage award too high: D will spend too many resources trying to avoid causing

harm. Damage award too low: D will have no incentive not to harm.

Problem: how do we set optimal damages when ex ante we don’t know what the harm will be?

Possible that damage awards don’t influence primary conduct of Ps very much, but they do influence Ds b/c awards are so large.

Easy cases: property damage, b/c P can recover market value. Hard cases: personal injuries, especially serious ones.

Tort theory of damages: put P back into position he was in before harm. With injuries and death, this can’t happen, so system has to find a way to approximate even

though money is insufficient replacement for life and health. Ergo: money damages are neither necessarily efficient, nor are they necessarily just, which

throws a wrench into the intended purpose of the tort system: to be just and efficient. Yet, goal of damages: compensation, deterrence, consistency. Compensable damages

Nonpecunary injuries, including both bodily harm and pain & suffering Medical expenses Lost earnings

Cover period before the trial and some estimate of future losses. Strategy is assessing damages: break down losses in order to come to some rational

measure. (Belli measure: have jury break down the person’s life into minutes and ask themselves how much the pain and suffering of each minute is worth—not favored in courts.) But how do you create a monetary value for pain and suffering

McDougald v Garber (NY 1989): woman made comatose vegetable by negligent administration of anesthesia. Issue: since P can’t enjoy life can she get damages for loss of enjoyment and pain & suffering? Held: to get these, there has to be some minimal level of awareness.

Losses in wrongful death don’t include medical expenses, pain & suffering, so how do we avoid undercompensating for death? In principle damages for death should be higher than that for serious injury, but they

aren’t. So tort system is creating an incentive to kill your victims have to bring in criminal justice to prevent this.

Epstein theory : go to first party insurance and scheduled damages, like in workers comp. Problem: tort system doesn’t allow K to waive or limit liability.

Scenario: high risk patient, can hospital K to cap damages? If not, then the rest of the population is subsidizing this patient. Prices go up, and people are drive out of the system. This is the problem with using a tort system rather than a workmen’s comp system in areas like health care where it is known ex ante that the risks are significant.

75